Final Adult Care 1

Ace your homework & exams now with Quizwiz!

leptin

secreted by fat cells; when abundant, causes brain to increase metabolism and decrease hunger

what are other ways to control bleeding in patients with Esophageal Varices

sengstaken-blackmore tube-balloon tamponade variceal ligation or banding endoscopic sclerosis endoscope

What is the most common method for testing/confirming H.pylori infection?

serologic testing

When should you administer H2 blockers?

single dose at *bedtime* for GI ulcers, heartburn, and PUD

Ghrelin

stimulates appetite gut hormone that regulates appetite through inhibition of leptin

Protein-rich meals, calcium, and vagus nerve excitation have what effect on acid secretion?

stimulatory effect

What type of mucosal lesion occurs after an acute medical crisis or trauma, such as sepsis or head injury?

stress ulcer

What drug is a *mucosal barrier fortifier* prescribed for patients with gastritis?

sucralfate

Which drug *binds with bile acids and pepsin to pepsin to protect stomach mucosa*?

sucralfate (*do not give within 30 min of giving antacids or other drugs*) (*give 1 hr before and 2 hr after meals and at bedtime*)

When should the patient take levothyroxine?

take an hour before meals on empty stomach

How should you assess for fluid volume deficit that occurs from bleeding?

take orthostatic BP and monitor for signs and symptoms of dehydration and dizziness

True or False: Chronic gastritis is associated with an increased risk for gastric cancer

true

True or False: Duodenal ulcers are deep and usually penetrate through the muscle layer of the stomach

true

True or False: Duodenal ulcers have a higher incidence in men but are increasing in women, especially post-menopausal women between 35-45 y.o

true

True or False: Food itself acts as an antacid by neutralizing stomach acid for 30-60 min

true

True or False: Gastric ulcers have a higher incidence in women over the age of 50

true

True or False: H.pylori can be tested from the patient's blood, breath, or stool

true

True or False: H.pylori is believed to be spread through contaminated food or water or contact with stool, vomit, saliva of an infected person

true

True or False: Hematemesis usually indicates bleeding at or above the duodenojejunal junction

true

True or False: If a patient has an esophageal stricture, it can be dilated during an EGD

true

True or False: If taken on an empty stomach, antacids are quickly evacuated and the neutralizing effect is reduced

true

True or False: Many herbs, such as powders of slippery elm, marshmalow root, quercetin, and licorice are commonly used by patients with gastritis and PUD

true

True or False: Mucosal changes that result from *acute gastritis* typically heal after several months

true

True or False: Patients with chronic gastritis may require injections of vitamin B12 for the treatment (or prevention) of pernicious anemia

true

True or False: Peptic ulcer perforation is a surgical emergency and can be life threatening

true

True or False: Side rails of hospital beds are raised after an EGD test while the sedation is wearing off

true

True or False: The expected outcome for PUD is that the patient is expected to report pain control as evidenced by no more than *3* on a scale of 0-10

true

True or false: Omeprazole may be dissolved in a sodium bicarbonate solution and given through any feeding tube & the capsule CAN be opened

true

True or False: A balanced diet, regular exercise, and stress-reduction techniques can help prevent gastritis

true (limiting: -caffeine -chocolate -mustard -pepper -hot spices -alcohol -tobacco) (stress reduction techniques: -aerobic exercise -meditation -reading -yoga)

True or false: Gastric lavage requires NG tube with instillation of a *room temp* solution in volumes of *200-300 ml*

true (no evidence whether sterile saline or sterile water is better than tap water)

Which type of *chronic gastritis* is nonerosive and refers to an inflammation of the *glands as well as the fundus and body of the stomach*?

type A chronic gastritis

Which type of chronic gastritis is *associated with the presence of antibodies to parietal cells and intrinsic factor*?

type A chronic gastritis (therefore an *autoimmune cause of this is likely*)

Which type of chronic gastritis usually affects the glands of the *antrum* but may involve the entire stomach?

type B chronic gastritis

What is the most common form of chronic gastritis and what is it caused by?

type B, H.pylori infection

<18.5

underweight BMI

<18.5

underwieght

After an EGD test, how long does the patient remain NPO?

until gag reflex returns (typically 30-60 min)

During gastric lavage, the solution and blood are repeatedly withdrawn manually until when?

until returns are *clear or light pink and without clots*

If the patient's respiratory rate falls below 10/min or if the exhaled CO2 level falls below 20% during an EGD, what does the nurse do?

uses a stimulus (*sternal rub*) to encourage deeper/faster respirations

what medications can we give to control bleeding in patients with Esophageal Varices (3)

vasopressin somatostatin octreotide

treatment of Esophageal Varices

ventilator or oxygen IV fluids blood products bleeding control

People with ___________ fat with ________________ obesity are at increased risk for cardiovascular disease and metabolic syndrome.

visceral truncal

less than 0.8 is optimal

waist to hip ratio

fat in upper legs

gynoid obesity

What causes atrophy of the thyroid gland?

hasimoto's grave's disease

manifestations of Esophageal Varices

hematemesis melena Progresses to hypovolemic shock

If perforation is present, what will the initial auscultation of bowel sounds be?

hyperactive, but diminishes as it progresses

The _____________ is a major site for regulating appetite.

hypothalamus

hyperplasia

increase in NUMBER of cells

hypertrophy

increase in cell SIZE

What is one of the main causes of acute gastritis?

infection of H.pylori

The hypothalamus, gut, and adipose tissue synthesize hormones and peptides that __________________ or ______________ appetite.

inhibit or stimulate

If GI bleeding is found during an EGD, what can the physician do?

inject sclerotherapy or other type agent to stop the bleeding

Obesity leads to increased in adipokines produced by fate cells these enzymes can lead to ________- and ___________.

insulin resistance and atherosclerosis

Sepsis is associated with what type of ulcers?

ischemic ulcers

What is the drug of choice for treating hypothyroidism?

levothyroxine

How should the patient be positioned during a gastric lavage?

lie on left side

NASH (non-alcoholic steatohepatitis)

lipids are deposited in the liver (causes fatty liver) can progress to cirrhosis

What is the endoscopy nurse's role during an EGD?

monitor rate, depth of respirations and O2 sat (shallow breaths decrease the amount of CO2 exhaled)

What is a complication of hypothyroidism?

myxedema coma

What are some patient education points for patients with hypothyroidism?

need for lifelong therapy don't switch brands unless prescribed need for warm enbironment because of cold intolerance avoid sedatives measures to minimize constipation such as increased fiber, increased activity, use of stool softeners

What does H.pylori do to the pH of stomach acid?

neutralizes it

18.5-24.9

normal body weight BMI

18.5-24.9

normal weight

What procedure involves injecting the patient with a contrast medium to scan for the presence of GI bleeding?

nuclear medicine scan

30 or above

obese BMI

25-29.9

overweight

25-29.9

overweight BMI

What is a mucosal lesion of the stomach or duodenum?

peptic ulcer

If your patient complains of sudden, sharp pain that begins in the midepigastric region then spreads over the entire abdomen, what most likely happened?

perforation

If the special carbon atom is found in the urea breath test, what does this finding indicate?

positive for H.pylori

what is endoscopic sclerosis

preventative measure where they inject something to harden the site so that it does not rupture

what is variceal ligation or banding

preventative therapy to prevent rupture -if patient is actively bleeding, it is hard for us to know where they are bleeding from

What is manifested by vomiting caused by stasis and gastric dilation?

pyloric obstruction (caused by scarring, edema, inflammation)

Aggressive acid suppression is used to prevent what?

rebleeding

what does octreotide do in a patient with Esophageal Varices

reduces the portal pressure

what does somatostatin do in a patient with Esophageal Varices

reduces the portal pressure

A nurse is providing dietary instructions to a client with a history of pancreatitis. Which instruction is correct? a) "Maintain a high-sodium, high-calorie diet." b) "Maintain a high-fat, high-carbohydrate diet." c) "Maintain a high-carbohydrate, low-fat diet." d) "Maintain a high-fat diet and drink at least 3 L of fluid a day."

"Maintain a high-carbohydrate, low-fat diet." A client with a history of pancreatitis should avoid foods and beverages that stimulate the pancreas, such as fatty foods, caffeine, and gas-forming foods; should avoid eating large meals; and should eat plenty of carbohydrates, which are easily metabolized. Therefore, the only correct instruction is to maintain a high-carbohydrate, low-fat diet. An increased sodium or fluid intake isn't necessary because chronic pancreatitis isn't associated with hyponatremia or fluid loss.

What is the *main feature of a duodenal ulcer*?

*high* gastric acid secretion (pH levels are low - excess acid)

Cardiovascular Clinical Manifestations of Hypothyroidism

- Decrease in cardiac contractility and output - ↑ Serum cholesterol and triglycerides - Anemia

GI Clinical Manifestations of Hypothyroidism

- Decreased appetite - Weight gain - N/V - Distended abdomen - Enlarged scaly tongue - Constipation - Celiac disease

Integumentary Clinical Manifestations of Hypothyroidism

- Dry, thick, inelastic, cold skin - Thick, brittle nails - Dry sparse coarse hair - Puffy face - Pallor - Decrease in sweating - Edema

Neurological Clinical Manifestations of Hypothyroidism

- Fatigue and lethargy - Personality and mood changes - Slowed speech

Musculoskeletal Clinical Manifestations of Hypothyroidism

- Muscular aches and pains - Slow movements - weakness

What is the approach to the management of chronic gastritis?

-*elimination of causative agents* (primary) -treatment of underlying disease -avoidance of toxic substances -health teaching

What are some common causes of *acute gastritis*?

-H.pylori -NSAIDs long-term -alcohol -coffee -caffeine -spicy foods -corticosteroids -radiation therapy -smoking -ingestion of corrosive substances

What are the causes of hypothyroidism?

-Iodine deficiency -Atrophy of the gland -Treatment for hyperthyroidism -Drugs -Cretinism if occurs in infancy

In regards to an EGD, what should you teach your patient?

-NPO 6-8 hrs before test -usual drug therapy can be taken morning of test (diabetics get special instructions) -avoid NSAIDs/anticoags for several days before the test -tell patient flexible tube will be passed down esophagus while under moderate sedation -anesthetic will suppress gag reflex so swallowing will be difficult -remove dentures -patient should have someone drive them home -patient should avoid driving 12-18 hours post EGD -hoarse voice/sore throat may persist for several days after test (may use throat lozenges) -may experience bruising around eyes which usually goes away in a week

Reproductive Clinical Manifestations of Hypothyroidism

-Prolonged menstrual periods or amenorrhea - Decreased libido, infertility

What are symptoms of pyloric obstruction?

-abdominal bloating -nausea -vomiting -metabolic alkalosis if vomiting persists -hypokalemia

What are some common causes of *chronic gastritis*?

-alcohol -radiation therapy -smoking -pyloric sphincter surgeries (reflux of alkaline secretions into the stomach) -Crohn's disease -graft vs. host disease -uremia

What are the key features associated with *upper GI bleeding*?

-bright red or coffee ground vomit -melena -decreased BP -increased HR -weak peripheral pulses -acute confusion -vertigo -dizziness/lightheadedness -syncope -decreased Hgb and Hct

What drugs are associated with gastritis episodes and should be avoided?

-corticosteroids -erthromycin -aspirin -NSAIDs

What are some common herbs and vitamins used for therapy in gastritis and PUD?

-cranberry -DGL -ginger -probiotics -slippery elm -vitamin C

Upper GI bleeding often requires the PCP or nurse to insert a large-bore NG tube to:

-determine presence or absence of blood in the stomach -assess the rate of bleeding -prevent gastric dilation -administer lavage

What are the key features associated with *acute gastritis*?

-epigastric pain/discomfort/tenderness -nausea & vomiting -hematemesis -melena -dyspepsia (heart burn) -anorexia -gastric hemorrhage -feeling of fullness

Atrophic gastritis is caused by what?

-exposure to toxis substances (benzene, lead, nickel) -H.pylori -autoimmune factors

What are the 3 types of ulcers associated with PUD?

-gastric -duodenal -stress

What are the most common complications of PUD?

-hemorrhage (*most serious*) -perforation -pyloric obstruction -intractable disease

How is perforation managed?

-immediately replacing fluid, blood, electrolytes -administering antibiotics -keeping the patient NPO -maintain nasogastric suction to drain gastric secretion and thus prevent further peritoneal spillage -monitor intake/output and check VS at least every hour -monitor for signs of septic shock (fever, pain, tachycardia, lethargy, anxiety)

Your patient presents with active upper GI bleeding. What should you do *first*?

-maintain ABC -start two large-bore IV lines for replacing fluids and blood -monitor VS, Hct, and O2 sat

What is the pathophysiology of gastritis (how does it occur)?

-prostaglandins provide a protective mucosal barrier that prevents the stomach from digesting itself ----> if there is a break in the protective barrier, *mucosal injury occurs* and the injury is *worsened by histamine release and vagus stimulation*. *hydrochloric acid* diffuses back into the mucosa and injures small vessels. -this back diffusion causes *edema*, *acute inflammatory cell infiltration*, and degenerative changes in the stomach lining

What is *chronic atrophic gastritis* characterized by?

-total loss of fundal glands -minimal inflammation -thinning of gastric mucosa -intestinal metaplasia *THESE CHANGES LEAD TO PUD AND GASTRIC CANCER*

What are the key features associated with *chronic gastritis*?

-upper abdominal discomfort -nausea & vomiting -anorexia -epigastric pain relieved by food -intolerance of fatty/spicy food -pernicious anemia

27. The nurse is preparing to administer a.m. medications to clients. Which medication should the nurse question before administering? 1. Pancreatic enzymes to the client who has finished breakfast. 2. The pain medication, morphine, to the client who has a respiratory rate of 20. 3. The loop diuretic to the client who has a serum potassium level of 3.9 mEq/L. 4. The beta blocker to the client who has an apical pulse of 68 bpm.

. 1. Rationale: Pancreatic enzymes must be administered with meals to enhance the digestion of starches and fats in the gastrointestinal tract

34. The client with an acute exacerbation of chronic pancreatitis has a nasogastric tube. Which interventions should the nurse implement? Select all that apply. 1. Monitor the client's bowel sounds. 2. Monitor the client's food intake. 3. Assess the client's intravenous site. 4. Provide oral and nasal care. 5. Monitor the client's blood glucose

. 1., 3, 4, 5 Rationale: 1 The return of bowel sounds indicates the return of peristalsis, and the nasogastric suction is usually discontinued within 24 to 48 hours thereafter Rationale: 3. The nurse should assess for signs of infection or infiltration. Rationale: 4. Fasting and the N/G tube increase the client's risk for mucous membrane irritation and breakdown. Rationale: 5. Blood glucose levels are monitored because clients with chronic pancreatitis can develop diabetes mellitus

ANS: B Because orlistat blocks the absorption of fatsoluble vitamins, the patient may not be receiving an adequate amount of vitamin K, resulting in a decrease in clotting factors. Abdominal bloating and liquid stools are common side effects of orlistat and indicate that the nurse should remind the patient that fat in the diet may increase these side effects. Weight loss plateaus are normal during weight reduction.

.16. The nurse obtains these assessment data for a patient who has been taking orlistat (Xenical) for several months as part of a weight loss program. Which finding is most important to report to the health care provider? a. The patient frequently has liquid stools. b. The patient is pale and has many bruises. c. The patient is experiencing a plateau in weight loss. d. The patient complains of abdominal bloating after meals.

A 15-year-old girl, the unrestrained passenger, in the front seat of a pickup truck was thrown from the vehicle when it hit a guard rail. She is admitted to the ER where she is evaluated by the ER physician. STAT lab work has been ordered and a student nurse asks why the doctor wants to have electrolytes done, "She's so young, and besides what abnormal labs could she have this early?" The best response would be: 1) "It is very possible that she will have an elevated potassium." 2) "We don't need to worry about what it will show; we'll just get it done." 3) "It is likely she will have an increased calcium due to her broken wrist." 4) "Electrolyte studies will indicate whether or not she had been drinking."

1) "It is very possible that she will have an elevated potassium."

A patient developed a bowel obstruction after surgery and a nasogastric tube was inserted. The most common electrolyte imbalances secondary to gastric suctioning is what? 1) Hyponatremia / hypokalemia 2) Hyponatremia / hyperkalemia 3) Hypernatremia / hypocalcemia 4) Hypernatremia / hypokalemia

1) Hyponatremia and Hypokalemia

An elderly patient has been ill with fever, muscle cramps, nausea and vomiting. His daughter calls the physician's office and asks what she should do. The nurse knows that these symptoms mostly likely represent what? 1) Hypovolemia 2) Hypocalcemia 3) Hypophosphatemia 4) Hypoaldosteronism

1) Hypovolemia

Which of the following are symptoms of hypovolemia? 1) oliguria 2) weight gain 3) decreased pulse and increased BP 4) distended jugular veins

1) Oliguria low urine output

Aldosterone promotes which of the following? 1) Reabsorption of sodium. 2) Reabsorption of potassium. 3) Excretion of renin. 4) Excretion of water.

1) Reabsorption of sodium.

Which of the following meals is highest in calcium? 1) Salmon sandwich with spinach and a glass of milk. 2) Fried egg sandwich with nuts. 3) Taco with tomatoes 4) Ham sandwich with pickles .

1) Salmon sandwich with spinach and a glass of milk.

Which of the following statements best describes electrolytes in extracellular and intracellular fluid? 1) There is a greater concentration of sodium in extracellular fluid and a greater concentration of potassium in intracellular fluid. 2) There is equal movement of sodium and potassium between intracellular and extracellular fluids. 3) There is a greater concentration of potassium in extracellular fluid and a greater concentration of sodium in intracellular fluid. 4) None of the above

1) There is a greater concentration of sodium in extracellular fluid and a greater concentration of potassium in intracellular fluid.

35. The nurse is administering a pancreatic enzyme to the client diagnosed with chronic pancreatitis. Which statement best explains the rationale for administering this medication? 1. It is an exogenous source of protease, amylase, and lipase. 2. This enzyme increases the number of bowel movements. 3. This medication breaks down in the stomach to help with digestion. 4. Pancreatic enzymes help break down fat in the small intestine

1. Rationale: Pancreatic enzymes enhance the digestion of starches (carbohydrates) in the gastrointestinal tract by supplying an exogenous (outside) source of the pancreatic enzymes protease, amylase, and lipase.

You are developing a care plan on Sally, a 67 y.o. patient with hepatic encephalopathy. Which of the following do you include? 1. Administering a lactulose enema as ordered. 2. Encouraging a protein-rich diet. 3. Administering sedatives, as necessary. 4. Encouraging ambulation at least four times a day.

1. Administering a lactulose enema as ordered.

You're caring for Betty with liver cirrhosis. Which of the following assessment findings leads you to suspect hepatic encephalopathy in her? 1. Asterixis 2. Chvostek's sign 3. Trousseau's sign 4. Hepatojugular reflex

1. Asterixis

You're caring for Jane, a 57 y.o. patient with liver cirrhosis who developed ascites and requires paracentesis. Before her paracentesis, you instruct her to: 1. Empty her bladder. 2. Lie supine in bed. 3. Remain NPO for 4 hours. 4. Clean her bowels with an enema.

1. Empty her bladder.

ANS: D Motivation is a key factor in successful weight loss and a shortterm outcome provides a higher motivation. A 21yearold patient is unlikely to be motivated by future health problems. Telling a patient that the initial weight loss is water will be discouraging, although this may be correct. Changing lifestyle habits is necessary, but this process occurs over time and discussing this is not likely to motivate the patient.

1. The nurse is developing a weight loss plan for a 21yearold patient who is morbidly obese. Which statement by the nurse is most likely to help the patient in losing weight on the planned 1000calorie diet? a. "It will be necessary to change lifestyle habits permanently to maintain weight loss." b. "You will decrease your risk for future health problems such as diabetes by losing weight now." c. "Most of the weight that you lose during the first weeks of dieting is water weight rather than fat." d. "You are likely to start to notice changes in how you feel with just a few weeks of diet and exercise."

environmental factors

1. greater access to food 2. increased portion sizes 3. lack of physical activity

What are the three ways in which a physician will stop the bleeding by endoscopic therapy via EGD?

1. inject chemicals into the bleeding site 2. treat bleeding area with heat, electric current, or laser 3. close the affected blood vessels with a band or a clip

The primary purposes of drug therapy in the treatment of PUD are what? (there's 4)

1. provide pain relief 2. eliminate H. pylori infection 3. heal ulcerations 4. prevent recurrence

ANS: B Elevated blood pressure is one of the characteristics of metabolic syndrome. The other information also may be obtained by the nurse, but it will not assist with the diagnosis of metabolic syndrome.

10. Which assessment will the nurse do to help determine if an obese patient seen in the clinic has metabolic syndrome? a. Take the patient's apical pulse. b. Check the patient's blood pressure. c. Ask the patient about dietary intake. d. Dipstick the patient's urine for protein.

ANS: A A fasting blood glucose test >100 mg/dL is one of the diagnostic criteria for metabolic syndrome. The other tests are not used to diagnose metabolic syndrome, although they may be used to check for cardiovascular complications of the disorder.

11. Which topic would be of importance for the nurse to include when teaching a patient about testing for possible metabolic syndrome? a. Blood glucose test b. Cardiac enzyme tests c. Postural blood pressures d. Resting electrocardiogram

ANS: C The restrictive nature of fad diets makes the weight loss achieved by the patient more difficult to maintain. Portion size can be estimated in other ways besides weighing. Severely calorierestricted diets are not necessary for patients in the overweight category of obesity and need to be closely supervised. Patients should weigh weekly rather than daily.

12. What specific information will the nurse include in patient teaching for an overweight patient who is starting a weight loss plan? a. Weigh yourself at the same time every morning. b. Start dieting with a 600 to 800calorie diet for rapid weight loss. c. Low carbohydrate diets lead to rapid weight loss but are difficult to maintain. d. Weighing all foods on a scale is necessary to choose appropriate portion sizes.

ANS: D The waisttohip ratio for this patient is 0.88, which exceeds the recommended level of <0.80. A BMI of 24 kg/m2 women with a waist circumference larger than 35 in (89 cm) and men with a waist circumference larger than 40 in (102 cm). A patient with a BMI of 18 kg/m2 considered underweight. is normal. Health risks associated with obesity increase in

13. Which of these patients in the clinic will the nurse plan to teach about risks associated with obesity? a. Patient who has a BMI of 18 kg/m2 b. Patient with a waist circumference 34 inches (86 cm) c. Patient who has a body mass index (BMI) of 24 kg/m2 d. Patient whose waist measures 30 in (75 cm) and hips measure 34 in (85 cm)

ANS: C NAP can assist with IV placement by assisting with patient positioning or holding skinfolds aside. Planning for care and patient teaching require RN level education and scope of practice.

14. Which nursing action included in the plan of care for a patient who is being admitted for bariatric surgery can the nurse delegate to nursing assistive personnel (NAP)? a. Demonstrate use of the incentive spirometer. b. Plan methods for bathing and turning the patient. c. Assist with IV insertion by holding adipose tissue out of the way. d. Develop strategies to provide privacy and decrease embarrassment.

ANS: C The initial nursing action should be assessment of any reason for the change in weight loss. The other actions may be needed, but further assessment is required before any interventions are planned or implemented.

15. A patient who has been successfully losing 1 lb weekly for several months is weighed at the clinic and has not lost any weight for the last month. The nurse should first a. review the diet and exercise guidelines with the patient. b. instruct the patient to weigh weekly and record the weights. c. ask the patient whether there have been any changes in exercise or diet patterns. d. discuss the possibility that the patient has reached a temporary weight loss plateau.

ANS: D The nurse should obtain information about the patient's perceptions of the reasons for the obesity to develop a plan individualized to the patient. The other information also will be obtained from the patient, but the patient is more likely to make changes when the patient's beliefs are considered in planning.

17. When developing a weight reduction plan for an obese patient who wants to lose weight, which question should the nurse ask first? a. "Which food types do you like best?" b. "How long have you been overweight?" c. "What kind of physical activities do you enjoy?" d. "What factors do you think led to your obesity?"

ANS: D Vomiting with an NG tube in place indicates that the NG tube needs to be repositioned by the surgeon to avoid putting stress on the gastric sutures. The nurse should implement actions to decrease skin irritation and have the patient cough and deep breathe, but these do not indicate a need for rapid notification of the surgeon. Frequent

18. On the first postoperative day the nurse is caring for a patient who has had a RouxenY gastric bypass procedure. Which assessment finding should be reported immediately to the surgeon? a. Use of patientcontrolled analgesia (PCA) several times an hour for pain b. Irritation and skin breakdown in skinfolds c. Bilateral crackles audible at both lung bases d. Emesis of bilecolored fluid past the nasogastric (NG) tube

ANS: C Coughing and deep breathing can prevent major postoperative complications such as carbon monoxide retention and hypoxemia. Information about passive range of motion, the NG tube, and postoperative modifications in lifestyle also will be discussed, but avoidance of respiratory complications is the priority goal after surgery.

19. In planning preoperative teaching for a patient undergoing a RouxenY gastric bypass as treatment for morbid obesity, the nurse places the highest priority on a. demonstrating passive rangeofmotion exercises to the legs. b. discussing the necessary postoperative modifications in lifestyle. c. teaching the patient proper coughing and deep breathing techniques. d. educating the patient about the postoperative presence of a nasogastric (NG) tube.

Which of the following is the normal level of potassium in the blood? 1) 1.5 to 4.0 2) 3.5 - 5.0 3) 5.0-7.5 4) none of the above

2) 3.5-5.0

Which of the following interventions would be the best choice to monitor fluid and electrolyte balance? 1) assess if client is voiding 2) assess daily weight 3) evaluate daily urine specimens 4) check daily sodium levels

2) Assess daily weight

26. Which client problem has priority for the client diagnosed with acute pancreatitis? 1. Risk for fluid volume deficit. 2. Alteration in comfort. 3. Imbalanced nutrition: less than body requirements. 4. Knowledge deficit

2. Rationale: Autodigestion of the pancreas results in severe epigastric pain, accompanied by nausea, vomiting, abdominal tenderness, and muscle guarding.

36. The client diagnosed with acute pancreatitis is being discharged home. Which statement by the client indicates the teaching has been effective? 1. "I should decrease my intake of coffee, tea, and cola." 2. "I will eat a low-fat diet and avoid spicy foods." 3. "I will check my amylase and lipase levels daily." 4. "I will return to work tomorrow but take it easy."

2. Rationale: High-fat and spicy foods stimulate gastric and pancreatic secretions and may precipitate an acute pancreatic attack.

ANS: A This selection is most consistent with the recommendation of the American Institute for Cancer Research that one third of the diet should be from animal sources and two thirds from plant source foods. The other choices all have higher ratios of animal origin foods to plant source foods than would be recommended.

2. After the nurse has completed teaching a patient about the recommended amounts of foods from animal and plant sources, which of these menu selections indicates that the initial instructions about diet have been understood? a. 3 oz of pork roast, a cup of corn, and a sliced tomato b. A chicken breast and a cup of tossed salad with nonfat dressing c.A 6 oz can of tuna mixed with nonfat mayonnaise and chopped celery d. 3 oz of roast beef, 2 oz of lowfat cheese, and a halfcup of carrot sticks

What is the primary concern of the nurse administering narcotics and sedatives to a client with cirrhosis? 1. The kidneys cannot excrete the drug. 2. The liver cannot metabolize drugs effectively. 3. Hepatic enzymes potentiate these drugs to high levels. 4. Narcotics and sedatives are contraindicated in clients with cirrhosis.

2. The liver cannot metabolize drugs effectively.

How long does an EGD procedure typically last?

20-30 min

Which of the following diets contain the most sodium? 1) Roast beef sandwich with mustard and an apple 2) Turkey sandwich with mashed potatoes 3) Bologna sandwich with pickles and potato chips 4) Veggie wrap with pears

3) Bologna sandwich with pickles and potato chips

When evaluating the effectiveness of IV fluid re-hydration for severe dehydration, which of the following is the best indicator? 1) Weight gain 2) Skin turgor 3) Increased urinary output 4) Increased temperature

3) Increased Urinary Output For a person with more acute or severe dehydration, you would expect to see increase in urine output and decrease in urine osmolarity with re-hydration.

Which of the following laboratory results places the patient at greatest risk for dysrhythmias? 1) Na+ 148 mEq/L 2) Ca++ 8.9 mg/dL 3) K+ 6.6 mEq/L 4) Na+ 133 mEq/L

3) K+ 6.6 mEq/dL Watch out for that Potassium! Normal is 3.5-5.0 (memorize this!!!!) Hyperkalemia can lead to cardiac dysrhythmias.

The preferred route for potassium replacement is: 1) IV push 2) IV Piggyback 3) Oral 4) Continuous drip

3) Oral Low potassium is most often seen in patients who have polyuria such as in diabetes or those on diuretics. An oral potassium supplement is usually prescribed and is the preferred route if tolerated.

A client is hyponatremic and the cause is thought to be fluid overload. When this is the etiology, what would be the intervention of choice? 1) restrict sodium intake 2) do nothing and allow it to resolve itself 3) restrict oral fluids 4) force fluids

3) Restrict oral fluids

A patient is receiving a loop diuretic. The nurse should be alert to which of the following symptoms? 1) Restlessness and agitation 2) Paresthesias and irritability 3) Weak, irregular pulse, cardiac arrhythmias, and muscle weakness 4) Increased blood pressure and muscle spasms

3) Weak, irregular pulse, cardiac arrhythmias, and muscle weakness

Which acid-base imbalance would be expected in a patient who has been having acute diarrhea for more than 24 hours? A. Metabolic acidosis B. Metabolic alkalosis C. Respiratory acidosis D. Respiratory alkalosis

A

31. The nurse is discussing complications of chronic pancreatitis with a client diagnosed with the disease. Which complication should the nurse discuss with the client? 1. Diabetes insipidus. 2. Crohn's disease. 3. Narcotic addiction. 4. Peritonitis.

3. Rationale: Narcotic addiction is related to the frequent, severe pain episodes often occurring with chronic pancreatitis which require narcotics for relief.

29. The nurse is completing discharge teaching to the client diagnosed with acute pancreatitis. Which instruction should the nurse discuss with the client? 1. Instruct the client to decrease alcohol intake. 2. Explain the need to avoid all stress. 3. Discuss the importance of stopping smoking. 4. Teach the correct way to take pancreatic enzymes.

3. Rationale: Smoking stimulates the pancreas to release pancreatic enzymes and should be stopped.

30. The male client diagnosed with chronic pancreatitis calls and reports to the clinic nurse he has been having a lot of "gas," along with frothy and very foul-smelling stools. Which intervention should the nurse implement? 1. Explain this is common for chronic pancreatitis. 2. Ask the client to bring in a stool specimen to the clinic. 3. Arrange an appointment with the HCP for today. 4. Discuss the need to decrease fat in the diet so this won't happen

3. Rationale: Steatorrhea (fatty, frothy, foul-smelling stool) is caused by a decrease in pancreatic enzyme secretion and indicates impaired digestion and possibly an increase in the severity of the pancreatitis. The client should see the HCP.

25. The client is admitted to the medical department with a diagnosis of R/O acute pancreatitis. Which laboratory values should the nurse monitor to confirm this diagnosis? 1. Creatinine and BUN. 2. Troponin and CK-MB. 3. Serum amylase and lipase. 4. Serum bilirubin and calcium.

3. Rationale: Serum amylase increases within two (2) to 12 hours of the onset of acute pancreatitis to two (2) to three (3) times normal and returns to normal in three (3) to four (4) days; lipase elevates and remains elevated for seven (7) to 14 days.

32. The client is immediate postprocedure endoscopic retrograde cholangiopancreatogram (ERCP). Which intervention should the nurse implement? 1. Assess for rectal bleeding. 2. Increase fluid intake. 3. Assess gag reflex. 4. Keep in supine position.

3. Rationale: The gag reflex will be suppressed as a result of the local anesthesia applied to the throat to insert the endoscope into the esophagus; therefore, the gag reflex must be assessed prior to allowing the client to resume eating or drinking.

33. The client diagnosed with acute pancreatitis is in pain. Which position should the nurse assist the client to assume to help decrease the pain? 1. Recommend lying in the prone position with legs extended. 2. Maintain a tripod position over the bedside table. 3. Place in side-lying position with knees flexed. 4. Encourage a supine position with a pillow under the knees.

3. Rationale: This fetal position decreases pain caused by the stretching of the peritoneum as a result of edema

Which arterial blood gas laboratory values would be seen in metabolic alkalosis? A. pH 7.49, bicarbonate 32 B. pH 7.28, carbon dioxide 54 C. pH 7.53, carbon dioxide 28 D. pH 7.31, bicarbonate 18

A

ANS: B Behavior modification programs focus on how and when the person eats and de emphasize aspects such as calorie counting. Nonfood rewards are recommended for achievement of weightloss goals. Patients are often taught to restrict eating to designated meals when using behavior modification.

3. When working with an obese patient who is enrolled in a behavior modification program, which nursing action is appropriate? a. Having the patient write down the caloric intake of each meal b. Asking the patient about situations that tend to increase appetite c. Encouraging the patient to eat small amounts throughout the day rather than having scheduled meals d. Suggesting that the patient have a reward, such as a piece of sugarless candy, after achieving a weightloss goal

A nurse should monitor blood glucose levels for a patient diagnosed with hyperinsulinism. What blood value does the nurse recognize as inadequate to sustain normal brain function? a) 90 mg/dL b) 50 mg/dL c) 30 mg/dL d) 70 mg/dL

30 mg/dL Hyperinsulinism is caused by overproduction of insulin by the pancreatic islets. Occasionally, tumors of nonpancreatic origin produce an insulinlike material that can cause severe hypoglycemia and may be responsible for seizures coinciding with blood glucose levels that are too low to sustain normal brain function

what percentage of US adults are obese?

34%

Average Waist Circumference

37.6-38.8 inches

The active process by which sodium moves out of the cell and potassium moves into the cell is called: 1) filtration. 2) osmosis. 3) diffusion. 4) active transport.

4) Active transport

Which of the following is responsible for the oncotic pressure in the blood stream? 1) Antidiuretic hormone 2) Blood urea nitrogen 3) Creatinine 4) Albumin

4) Albumin

Which of the following are symptoms of hypocalcemia (low calicum)? 1) Positive Chvostek's and Trousseau signs and seizures 2) Prolonged QT interval 3) Numbness and tingling of the hands and muscle cramps 4) All of the above

4) All of the above

28. The client is diagnosed with acute pancreatitis. Which health-care provider's admitting order should the nurse question? 1. Bedrest with bathroom privileges. 2. Initiate IV therapy of D5W at 125 mL/hr. 3. Weigh client daily. 4. Low-fat, low-carbohydrate diet.

4. Rationale: The client will be NPO, which will decrease stimulation of the pancreatic enzymes, resulting in decreased autodigestion of the pancreas, therefore decreasing pain.

ANS: A Exercise should be done daily for 30 minutes to an hour. Exercising in highly aerobic activities for short bursts or only once a week is not helpful and may be dangerous in an individual who has not been exercising. Running may be appropriate, but a patient should start with an exercise that is less stressful and can be done for a longer period. Weight lifting is not as helpful as aerobic exercise in weight loss.

4. Which patient behavior indicates that an overweight patient has understood the nurse's teaching about the best exercise plan for weight loss? a. Walking for 40 minutes 6 or 7 days/week b. Lifting weights with friends 3 times/week c. Playing soccer for an hour on the weekend d. Running for 10 to 15 minutes 3 times/week

A patient with advanced cirrhosis with ascites is short of breath and has an increased respiratory rate. The nurse should 1. initiate oxygen therapy at 2 L/min to increase gas exchange. 2. notify the health care provider so a paracentesis can be performed. 3. ask patient to cough and deep breathe to clear respiratory secretions. 4. place the patient in Fowler's position to relieve pressure on the diaphragm.

4. place the patient in Fowler's position to relieve pressure on the diaphragm.

ANS: C Side effects of sibutramine (Meridia) include hypertension. A permanent pacemaker and a history of fenfluramine use are not contraindications for sibutramine. Sibutramine is prescribed for patients who have large weight loss goals. DIF: Cognitive Level: Application REF: 953954

5. When the health care provider in the outpatient clinic is considering prescribing sibutramine (Meridia) for a patient, which patient information is most important for the nurse to discuss with the provider? a. The patient has a permanent pacemaker. b. The patient's goal is to lose 90 lb (41 kg). c. The patient's blood pressure is usually 135145/8595. d. The patient used fenfluramine (Pondimin) in the past for weight loss.

Estimates of obesity as a inherited problem are more than ___%

50

ANS: B The patient may be experiencing an increase in heart rate caused by the sibutramine (Meridia) that should be evaluated further by the health care provider. Plateaus during weight loss programs are common. Chronic constipation may be a side effect of the sibutramine, and the nurse should instruct the patient in measures such as eating more high fiber foods and increasing fluid intake. The nurse should reinforce the need to exercise more frequently, but no additional intervention by the health care provider is necessary regarding the patient's activity level.

6. A patient has been on a 1000calorie diet with a daily exercise routine and a prescription for sibutramine (Meridia) for 10 weeks. Which information obtained by the nurse is important to report to the health care provider? a. The patient has not lost any weight for the last 2 weeks. b. The patient tells the nurse about occasional palpitations. c. The patient complains about having chronic constipation. d. The patient reports walking only 3 days during the last week.

ANS: D Reconstructive surgery may be used to eliminate excess skinfolds after at least a year has passed since the surgery. Skinfolds may not disappear over time, especially in older patients. The response, "The important thing is that your weight loss is improving your health" ignores the patient's concerns about appearance and implies that the nurse knows what is important. Whereas it may be helpful for the patient to talk to a counselor, it is more likely to be helpful to know that cosmetic surgery is available.

7. A few months after bariatric surgery, a 62yearold patient tells the nurse, "My skin is hanging in folds. I think I need cosmetic surgery." Which response by the nurse is most appropriate? a. "Perhaps you would like to talk to a counselor about your body image." b. "The important thing is that your weight loss is improving your health." c. "The skinfolds will gradually disappear once most of the weight is lost." d. "Cosmetic surgery is certainly a possibility once your weight has stabilized."

ANS: D The incision should be protected from strain to decrease the risk for wound dehiscence. The patient should be encouraged to use the PCA since pain control will improve cough effort and patient mobility. NG irrigation may damage the suture line or overfill the stomach pouch. Sugarfree clear liquids are offered during the immediate postoperative time to decrease the risk for dumping syndrome.

8. A patient returns to the surgical nursing unit following a vertical banded gastroplasty with a nasogastric tube to low, intermittent suction and a patientcontrolled analgesia (PCA) machine for pain control. Which nursing action should be included in the postoperative plan of care? a. Irrigate the nasogastric (NG) tube frequently with normal saline. b. Offer sips of sweetened liquids at frequent intervals. c. Remind the patient that PCA use may slow the return of bowel function. d. Support the surgical incision during patient coughing and turning in bed.

.ANS: A Intake of fluids with meals tends to cause dumping syndrome and diarrhea. Food choices should be low in fat and fiber. Exercise does not prevent the development of flabby skin.

9. Which information will the nurse plan to include in discharge teaching for a patient after gastric bypass surgery? a. Avoid drinking fluids with meals. b. Choose highfat foods for at least 30% of intake. c. Choose foods that are high in fiber to promote bowel function. d. Development of flabby skin can be prevented by daily exercise.

A charge nurse is teaching a group of nurses about conditions related to metabolic acidosis. Which of the following statements by a unit nurse indicated the teaching has been effective? A. "Metabolic acidosis can occur due to diabetic ketoacidosis" B. "Metabolic acidosis can occur in a client who has myasthenia gravis." C. "Metabolic acidosis can occur in a client who has asthma" D. "Metabolic acidosis can occur due to cancer."

A

A nurse is assessing a client who has hyperkalemia. The nurse should identify which of the following conditions as being associates with this electrolyte imbalance? A. Diabetic ketoacidosis B. Heart failure C. Cushing's syndrome D. Thyroidectomy

A

A nurse is caring for a client who has a blood potassium of 5.4 mEq/L. The nurse should assess for which of the following manifestations? A. ECG changes B. Constipation C. Polyuria D. Paresthesia

A

A nurse is caring for a patient who has a blood sodium level 133 mEq/L and blood potassium level 3.4 mEq/L. The nurse should recognize that which of the following treatments can results in these laboratory findings? A. Three tap water enemas B. 0.9% sodium chloride solution IV at 50ml/hr C. 5% dextrose with 0.45% sodium chloride solution with 20 mEq of K+ IV at 80 mL/hr D. Antibiotic therapy

A

A patient has a low serum potassium level and is prescribed a dose of parenteral potassium chloride (KCl). Which administration method would the nurse use? A. Infuse 10 mEq over a 1-hour period B. Administer 5 mEq IM. C. Push 5 mEq through a central access line D. Dilute 200 mEq in 1L of normal saline, and infused at 100ml/hr.

A

A patient reports painful muscles spasms in the lower legs at rest, a tingling sensation in the hands and lips, and abdominal cramping and diarrhea. Which disorder would the nurse suspect? A. Hypocalcemia B. Hypernatremia C. Hypermagnesemia D. Hypophosphatemia

A

Deep and rapid breaths consistent with Kussmaul respiration are found in patients with which type of acid-base imbalance A. Metabolic acidosis B. Metabolic alkalosis C. Respiratory acidosis D. Respiratory alkalosis

A

How does the corresponding increase in carbon dioxide levels that occurs when arterial pH drops assist in maintaining acid-base balance? A. Carbon dioxide loss through exhalation can raise arterial pH levels. B. Carbon dioxide retention during exhalation can lower arterial pH levels. C. Carbon dioxide is a base that can convert free hydrogen ions into a neutral substance. D. Carbon dioxide is a buffer that can bind free hydrogen ions and form a neutral substance.

A

The healthcare provider writes prescriptions for a patient who is admitted with a serum potassium (K) level of 6.9 mEq/L. Which action would the nurse implement first? A. Place the patient on a cardiac monitor B. Administer sodium polystyrene sulfonate orally C. Ensure that a potassium-restricted diet is prescribed D. Teach the patient about foods that are high in potassium.

A

The primary health care provider prescribes IV administration of 100 ml of 20% glucose along with 20 unites of insulin. Which condition would the nurse expect the patient to have? A. Hyperkalemia B. Hyperglycemia C. Hypernatremia D. Hypercalcemia

A

Which assessment has the highest priority for the nurse to perform for a client with syndrome of inappropriate antidiuretic hormone (SIADH) receiving tolvaptan therapy for 24 hours? A. Evaluating serum sodium levels B. Evaluating serum potassium levels C. Examining the skin and sclera for jaundice D. Examining the IV site for indications of phlebitis

A

Which assessment would the nurse make immediately if a patient's serum potassium is reported to be 5.6 mEq/L? A. Heart rate B. Bowel sounds C. Paresthesia D. Recent diarrhea

A

Which intervention would the nurse plan for a patient with hypercalcemia? A. Monitor cardiac rhythm for changes B. Limit activities to protect against injury C. Assess oxygen saturation levels every 4 hours D. Avoid invasive procedures because of increased bleeding risk

A

Which patient would be appropriate to assign to the new nurse working on the medical-surgical unit? A. Patient with COPD with a partial pressure of arterial carbon dioxide level of 50 mm hg B. Patient with reactive airway disease, wheezing, and partial pressure of arterial oxygen level of 62 mm Hg C. Patient with diabetic ketoacidosis and change in mental status who has a pH of 7.18 D. Patient with an irregular heart rate and prolonged vomiting with bicarbonate level of 40 mEq/L

A

Which physiologic imbalance is the patient at risk for developing if he or she has chronically low hemoglobin? A. Acidosis B. Alkalosis C. Hypokalemia D. Ineffective ventilation

A

A patient with suspected adrenal insufficiency has been ordered an adrenocorticotropic hormone (ACTH) stimulation test. Administration of ACTH caused a marked increase in cortisol levels. How should the nurse interpret this finding? A) The patients pituitary function is compromised. B) The patients adrenal insufficiency is not treatable. C) The patient has insufficient hypothalamic function. D) The patient would benefit from surgery

A (An adrenal response to the administration of a stimulating hormone suggests inadequate production of the stimulating hormone. In this case, ACTH is produced by the pituitary and, consequently, pituitary hypofunction is suggested. Hypothalamic function is not relevant to the physiology of this problem. Treatment exists, although surgery is not likely indicated.)

A patient with thyroid cancer has undergone surgery and a significant amount of parathyroid tissue has been removed. The nurse caring for the patient should prioritize what question when addressing potential complications? A) Do you feel any muscle twitches or spasms? B) Do you feel flushed or sweaty? C) Are you experiencing any dizziness or lightheadedness? D) Are you having any pain that seems to be radiating from your bones?

A (As the blood calcium level falls, hyperirritability of the nerves occurs, with spasms of the hands and feet and muscle twitching. This is characteristic of hypoparathyroidism. Flushing, diaphoresis, dizziness, and pain are atypical signs of the resulting hypocalcemia.)

A nurse works in a walk-in clinic. The nurse recognizes that certain patients are at higher risk for different disorders than other patients. What patient is at a greater risk for the development of hypothyroidism? A) A 75-year-old female patient with osteoporosis B) A 50-year-old male patient who is obese C) A 45-year-old female patient who used oral contraceptives D) A 25-year-old male patient who uses recreational drugs

A (Even though osteoporosis is not a risk factor for hypothyroidism, the condition occurs most frequently in older women.)

The nurse is caring for a patient diagnosed with hypothyroidism secondary to Hashimotos thyroiditis. When assessing this patient, what sign or symptom would the nurse expect? A) Fatigue B) Bulging eyes C) Palpitations D) Flushed skin A (Symptoms of hypothyroidism include extreme fatigue, hair loss, brittle nails, dry skin, voice huskiness or hoarseness, menstrual disturbance, and numbness and tingling of the fingers. Bulging eyes, palpitations, and flushed skin would be signs and symptoms of hyperthyroidism.)

A (Symptoms of hypothyroidism include extreme fatigue, hair loss, brittle nails, dry skin, voice huskiness or hoarseness, menstrual disturbance, and numbness and tingling of the fingers. Bulging eyes, palpitations, and flushed skin would be signs and symptoms of hyperthyroidism.)

You are developing a care plan for a patient with Cushing syndrome. What nursing diagnosis would have the highest priority in this care plan? A) Risk for injury related to weakness B) Ineffective breathing pattern related to muscle weakness C) Risk for loneliness related to disturbed body image D) Autonomic dysreflexia related to neurologic changes

A (The nursing priority is to decrease the risk of injury by establishing a protective environment. The patient who is weak may require assistance from the nurse in ambulating to prevent falls or bumping corners or furniture. The patients breathing will not be affected and autonomic dysreflexia is not a plausible risk. Loneliness may or may not be an issue for the patient, but safety is a priority.)

Symptoms of hyperthyroidism

Systolic hypertension, atrial fibrillation, increased appetite, weight loss, warm smooth moist skin, fatigue, muscle weakness and Tremors insomnia, heat intolerance

C (Cushing syndrome causes characteristic physical changes that are likely to result in disturbed body image. Decisional conflict and powerless may exist, but disturbed body image is more likely to be present. Cognitive changes take place in patients with Cushing syndrome, but these may or may not cause spiritual distress.)

A 30 year-old female patient has been diagnosed with Cushing syndrome. What psychosocial nursing diagnosis should the nurse most likely prioritize when planning the patients care? A) Decisional conflict related to treatment options B) Spiritual distress related to changes in cognitive function C) Disturbed body image related to changes in physical appearance D) Powerlessness related to disease progression

A (Feedback can be either negative or positive. Most hormones are secreted in response to negative feedback; a decrease in levels stimulates the releasing gland.)

A client has a decreased level of thyroid hormone being excreted. What will the feedback loop do to maintain the level of thyroid hormone required to maintain homeostatic stability? a) Stimulate more hormones using the negative feedback system b) The feedback loop will be unable to perform in response to low levels of thyroid hormone. c) Produce a new hormone to try and regulate the thyroid function d) Stimulate more hormones using the positive feedback system

D (Hypoparathyroidism may slow bone resorption, reduce the serum calcium level, and cause profound neuromuscular irritability (as evidenced by tetany).)

A client is admitted to an acute care facility with a tentative diagnosis of hypoparathyroidism. The nurse should monitor the client closely for the related problem of: a) excessive thirst. b) acute gastritis. c) severe hypotension. d) profound neuromuscular irritability.

A (Hypothyroidism markedly decreases the metabolic rate, causing a reduced body temperature and cold intolerance. Other signs and symptoms include dyspnea, hypoventilation, bradycardia, hypotension, anorexia, constipation, decreased intellectual function, and depression.)

A client is being evaluated for hypothyroidism. During assessment, the nurse should stay alert for: a) decreased body temperature and cold intolerance. b) flushed, warm, moist skin. c) exophthalmos and conjunctival redness. d) systolic murmur at the left sternal border.

C (Urine output may be as high as 20 L/24 hours. Urine is dilute, with a specific gravity of 1.002 or less. Limiting fluid intake does not control urine exertion. Thirst is excessive and constant. Activities are limited by the frequent need to drink and void. Weakness, dehydration, and weight loss develop.)

A client sustained a head injury when falling off of a ladder. While in the hospital, the client begins voiding large amounts of clear urine and states he is very thirsty. The client states that he feels weak and has had an 8-lb weight loss since admission. What should the client be tested for? a) Hypothyroidism b) Syndrome of inappropriate antidiuretic hormone secretion (SIADH) c) Diabetes insipidus (DI) d) Pituitary tumor

D (Tetany may result if the parathyroid glands are excised or damaged during thyroid surgery.)

A nurse is assessing a client after a thyroidectomy. The assessment reveals muscle twitching and tingling, along with numbness in the fingers, toes, and mouth area. The nurse should suspect which complication? a) Laryngeal nerve damage b) Hemorrhage c) Thyroid storm d) Tetany

B (The adrenal medulla secretes epinephrine and norepinephrine.)

A group of students are reviewing material about endocrine system function. The students demonstrate understanding of the information when they identify which of the following as secreted by the adrenal medulla? a) Glucagon b) Epinephrine c) Mineralocorticoids d) Glucocorticoids

A (Weight loss, nervousness, and tachycardia are signs of hyperthyroidism. Other signs of hyperthyroidism include exophthalmos, diaphoresis, fever, and diarrhea.)

A nurse is assessing a client with hyperthyroidism. What findings should the nurse expect? a) Weight loss, nervousness, and tachycardia b) Weight gain, constipation, and lethargy c) Exophthalmos, diarrhea, and cold intolerance d) Diaphoresis, fever, and decreased sweating

C (Hypothalamic dopamine inhibits the release of prolactin from the anterior pituitary gland.)

A nurse educator is teaching a chapter on, "The Function of the Endocrine System." Which of the following hormones would she not include as one of the six hypothalamic hormones? a) Corticotropin-releasing hormone b) Thyrotropin-releasing hormone c) Prolactin d) Gonadotropin-releasing hormone

B (The client requires additional teaching if he states that he will increase his calcium intake. Hyperparathyroidism causes extreme increases in serum calcium levels. The client should increase his fluid intake, but he should limit his calcium and vitamin D intake.)

A nurse is caring for a client who was recently diagnosed with hyperparathyroidism. Which statement by the client indicates the need for additional discharge teaching? a) "I'll schedule a follow-up visit with my physician as soon as I get home." b) "I will increase my fluid and calcium intake." c) "I'll call my physician if I notice tingling around my lips." d) "I will take my pain medications according to the schedule we developed."

D (An acute addisonian crisis is a life-threatening event, caused by deficiencies of cortisol and aldosterone. Glucocorticoid insufficiency causes a decrease in cardiac output and vascular tone, leading to hypovolemia. The client becomes tachycardic and hypotensive and may develop shock and circulatory collapse.)

A nurse is planning care for a client in acute addisonian crisis. Which nursing diagnosis should receive the highest priority? a) Imbalanced nutrition: Less than body requirements b) Risk for infection c) Impaired physical mobility d) Decreased cardiac output

D (The thyroid gland is located in the lower neck, anterior to the trachea.)

A nurse is preparing to palpate the thyroid gland. Where would the nurse expect to find this gland? a) In the abdomen, directly above the kidneys b) In the upper part of the chest near the heart c) In the right to left upper quadrant of the abdomen d) In the lower neck, anterior to the trachea

B (The adrenal cortex manufactures and secretes glucocorticoids, such as cortisol, which affect body metabolism, suppress inflammation, and help the body withstand stress. The adrenal cortex manufactures and secretes cortisol.)

A nurse is reviewing the laboratory order for a client suspected of having an endocrine disorder. The lab slip includes obtaining cortisol levels. Which of the following is being tested? a) Parathyroid functioning b) Adrenal functioning c) Thymus functioning d) Thyroid functioning

A (The client requires additional teaching because he states that he may stop taking corticosteroids when he feels better. Corticosteroids should be gradually tapered by the physician. Tapering the corticosteroid allows the adrenal gland to gradually resume functioning.)

A nurse is teaching a client with adrenal insufficiency about corticosteroids. Which statement by the client indicates a need for additional teaching? a) "I may stop taking this medication when I feel better." b) "I will eat lots of chicken and dairy products." c) "I will see my ophthalmologist regularly for a check-up." d) "I will avoid friends and family members who are sick."

B (The nurse should explain to the client that Cushing's syndrome causes physical changes related to excessive corticosteroids.)

A nurse should perform which intervention for a client with Cushing's syndrome? a) Suggest a high-carbohydrate, low-protein diet. b) Explain that the client's physical changes are a result of excessive corticosteroids. c) Offer clothing or bedding that's cool and comfortable. d) Explain the rationale for increasing salt and fluid intake in times of illness, increased stress, and very hot weather.

B (Excess catecholamine release occurs with pheochromocytoma and causes hypertension. The nurse should prepare to administer nitroprusside to control the hypertension until the client undergoes adrenalectomy to remove the tumor.)

A nursing coordinator calls the intensive care unit (ICU) to inform the department that a client with a suspected pheochromocytoma will be admitted from the emergency department. The ICU nurse should prepare to administer which drug to the client? a) Insulin b) Nitroprusside c) Dopamine (Inotropin) d) Lidocaine

B, D (Thyroid storm necessitates interventions to reduce heart rate and temperature. Diuretics, insulin, and steroids are not indicated to address the manifestations of this health problem.)

A patient has been admitted to the critical care unit with a diagnosis of thyroid storm. What interventions should the nurse include in this patients immediate care? Select all that apply. A) Administering diuretics to prevent fluid overload B) Administering beta blockers to reduce heart rate C) Administering insulin to reduce blood glucose levels D) Applying interventions to reduce the patients temperature E) Administering corticosteroids

C, E (Patients with aldosteronism exhibit a profound decline in the serum levels of potassium, and hypertension is the most prominent and almost universal sign of aldosteronism. Pupillary response, peripheral pulses, and renal function are not directly affected.)

A patient has been assessed for aldosteronism and has recently begun treatment. What are priority areas for assessment that the nurse should frequently address? Select all that apply. A) Pupillary response B) Creatinine and BUN levels C) Potassium level D) Peripheral pulses E) BP

D (Corticosteroid dosages are reduced gradually (tapered) to allow normal adrenal function to return and to prevent steroid-induced adrenal insufficiency. There are no OTC substitutes for prednisone and neither calcium chloride nor levothyroxine addresses the risk of adrenal insufficiency.)

A patient has been taking prednisone for several weeks after experiencing a hypersensitivity reaction. To prevent adrenal insufficiency, the nurse should ensure that the patient knows to do which of the following? A) Take the drug concurrent with levothyroxine (Synthroid). B) Take each dose of prednisone with a dose of calcium chloride. C) Gradually replace the prednisone with an OTC alternative. D) Slowly taper down the dose of prednisone, as ordered.

A (Therapeutic use of corticosteroids is the most common cause of adrenocortical insufficiency. The other options also cause adrenocortical insufficiency, but they are not the most common causes.)

A patient is undergoing testing for suspected adrenocortical insufficiency. The care team should ensure that the patient has been assessed for the most common cause of adrenocortical insufficiency. What is the most common cause of this health problem? A) Therapeutic use of corticosteroids B) Pheochromocytoma C) Inadequate secretion of ACTH D) Adrenal tumor

D (Suppression of the adrenal cortex may persist up to 1 year after a course of corticosteroids of only 2 weeks duration.)

A patient on corticosteroid therapy needs to be taught that a course of corticosteroids of 2 weeks duration can suppress the adrenal cortex for how long? A) Up to 4 weeks B) Up to 3 months C) Up to 9 months D) Up to 1 year

B (Muscle wasting can be partly addressed through increased protein intake. Passive ROM exercises maintain flexibility, but do not build muscle mass. Vitamin D and calcium supplements do not decrease muscle wasting. Activity limitation would exacerbate the problem.)

A patient who has been taking corticosteroids for several months has been experiencing muscle wasting. The patient has asked the nurse for suggestions to address this adverse effect. What should the nurse recommend? A) Activity limitation to conserve energy B) Consumption of a high-protein diet C) Use of OTC vitamin D and calcium supplements D) Passive range-of-motion exercises

A (Before, during, and after this surgery, blood glucose monitoring and assessment of stools for blood are carried out. The patients blood sugar is more likely to be volatile than body weight or temperature. Hematuria is not a common complication.)

A patient with Cushing syndrome as a result of a pituitary tumor has been admitted for a transsphenoidal hypophysectomy. What would be most important for the nurse to monitor before, during, and after surgery? A) Blood glucose B) Assessment of urine for blood C) Weight D) Oral temperature

A, C, D (Foods high in vitamin D, protein, and calcium are recommended to minimize muscle wasting and osteoporosis. Referral to a dietitian may assist the patient in selecting appropriate foods that are also low in sodium and calories.)

A patient with Cushing syndrome has been hospitalized after a fall. The dietician consulted works with the patient to improve the patients nutritional intake. What foods should a patient with Cushing syndrome eat to optimize health? Select all that apply. A) Foods high in vitamin D B) Foods high in calories C) Foods high in protein D) Foods high in calcium E) Foods high in sodium

C (Myxedema coma is a rare life-threatening condition. It is the decompensated state of severe hypothyroidism in which the patient is hypothermic and unconscious (Ross, 2012a). This condition may develop with undiagnosed hypothyroidism and may be precipitated by infection or other systemic disease or by use of sedatives or opioid analgesic agents. Patients may also experience myxedema coma if they forget to take their thyroid replacement medication. (less))

A patient with a history of hypothyroidism is admitted to the intensive care unit unconscious and with a temperature of 95.2ºF. A family member informs the nurse that the patient has not taken thyroid medication in over 2 months. What does the nurse suspect that these findings indicate? a) Syndrome of inappropriate antidiuretic hormone (SIADH) b) Diabetes insipidus c) Myxedema coma d) Thyroid storm

D (In all patients with hypothyroidism, the effects of analgesic agents, sedatives, and anesthetic agents are prolonged. There is no direct increase in the risk of anaphylaxis, nausea, or drug interactions, although these may potentially result from the prolonged half-life of drugs.)

A patient with a recent diagnosis of hypothyroidism is being treated for an unrelated injury. When administering medications to the patient, the nurse should know that the patients diminished thyroid function may have what effect? A) Anaphylaxis B) Nausea and vomiting C) Increased risk of drug interactions D) Prolonged duration of effect

D (IV administration of corticosteroids (methylprednisolone sodium succinate [Solu-Medrol]) may begin on the evening before surgery and continue during the early postoperative period to prevent adrenal insufficiency. Antibiotics, antihypertensives, and parenteral nutrition do not prevent adrenal insufficiency or other common complications of adrenalectomy.)

A patient with pheochromocytoma has been admitted for an adrenalectomy to be performed the following day. To prevent complications, the nurse should anticipate preoperative administration of which of the following? A) IV antibiotics B) Oral antihypertensives C) Parenteral nutrition D) IV corticosteroids

Which nursing intervention is consistent with safe administration of IV potassium to a patient with hypokalemia? Select all that apply, letter only. A. Evaluate the heart rate and regularity B. Establish and evaluate the patency of a large vein C. Obtain an IV controller device (pump) D. Plan to assess the respiratory rate and oxygen saturation every hour E. Prepare to administer potassium IV push to reduce the risk for infiltration F. Encourage the patient to ambulate independently to relieve muscle cramps.

A, B, C, D

Although it is necessary to assess patient history of RENAL disease before administering antacids, which antacid is okay to give to patients WITH renal disease?

aluminum hydroxide

5. Which of the following interventions would be most appropriate for a client who has urge incontinence? A) Have the client urinate on a timed schedule. B) Provide a bedside commode. C) Administer prophylactic antibiotics. D) Teach the client intermittent self-catheterization technique. E) Have the client urinate on a timed schedule

A) Have the client urinate on a timed schedule.

8. The catheter slips into the vagina during a straight catheterization of a female client. The nurse does which action? A) Leaves the catheter in place and gets a new sterile catheter. B) Leaves the catheter in place and asks another nurse to attempt the procedure. C) Removes the catheter and redirects it to the urinary meatus. D) Removes the catheter, wipes it with a sterile gauze, and redirects it to the urinary meatus

A) Leaves the catheter in place and gets a new sterile catheter.

7. During assessment of the client with urinary incontinence, the nurse is most likely to assess for which of the following? Select all that apply. A) Perineal skin irritation B) Fluid intake of less than 1,500 mL/day C) History of antihistamine intake D) History of frequent urinary tract infections E) A fecal impaction

A) Perineal skin irritation B) Fluid intake of less than 1,500 mL/day D) History of frequent urinary tract infections E) A fecal impaction

A nurse is admitting a client who reports nausea, vomiting, and weakness. The client has dry oral mucous membranes and blood pressure 102/64 mm Hg. which of the following findings should the nurse identify as manifestations of fluid volume deficit? Select all that apply, letter only A. Decreased skin turgor B. Concentrated urine C. Bradycardia D. Low-grade fever E. Tachypnea

A, B, D, E

A nurse is admitting an older adult client who reports a weight gain of 2.3 kg (5lb) in 48 hours. Which is the following manifestations of fluid volume excess should the nurse expect? Select all that apply, letter only. A. Dyspnea B. Edema C. Bradycardia D. Hypertension E. Weakness

A, B, D, E

A 30-year-old male client having an annual health physical reports that all of the following changes have developed during the past year. Which ones alert the nurse to possible pituitary hyperfunction. Select all that apply letter only. A. 15lb weight gain B. Decreased libido C. Four sinus infections D. Frequent constipation E. Increased foot callus formation F. Occasional dripping of clear fluid from both breasts G. Severely sprained angle from a volleyball injury

A, B, F

Which clinical indicators are most relevant for the nurse to monitor during IV fluid replacement for a client with dehydration? Select all that apply, letter only. A. Blood pressure B. Deep tendon reflexes C. Hand-grip strength D. Pulse rate and quality E. Skin turgor F. Urine output

A, D, F

The nurse is reviewing the history of a patient with suspected of having hyperthyroidism. Which manifestation(s) would be supportive of the diagnosis? (Select all that apply.) A. Hyperactivity with increasing sense of fatigue B. Increase in appetite C. Emotional instability D. Mental sluggishness E. Heat intolerance

A,B,C,E

Which patient is at a potential risk for Digoxin toxicity? A. A patient with Cushing's syndrome taking Laxis 20 mg IV twice a day B. A patient with a calcium level of 8.9 C. A patient with a potassium level of 3.8 D. A patient presenting with painful muscle spasms and positive Trousseau's sign

A. A patient with Cushing's syndrome taking Laxis 20 mg IV twice a day

Which patient below is considered hypernatremic? A. A patient with a sodium level of 155 B. A patient with a sodium level of 145 C. A patient with a sodium level of 120 D. A patient with a sodium level of 136

A. A patient with a sodium level of 155

Which patient below is considered hypernatremic?* A. A patient with a sodium level of 155 B. A patient with a sodium level of 145 C. A patient with a sodium level of 120 D. A patient with a sodium level of 136

A. A patient with a sodium level of 155

A patient with acromegaly will most likely exhibit which symptoms: A. Bone pain B. Frequent infections C. Fatigue D. Weight loss

A. Bone pain Acromegaly results from increased secretion of growth hormone, causing elongation and expansion of the bones. Infection and weight loss are not directly associated with this disorder. Although the disease process causes patients to become fatigued, the best answer is bone pain.

A client with cirrhosis is at risk for developing complications. Which condition is the most serious and potentially life-threatening? A. Esophageal varices B. Asterixis (liver flap) C. Peripheral edema D. Ascites

A. Esophageal varices

Magnesium is absorbed by what system of the body? A. Gastrointestinal B. Hepatic C. Lymphatic D. Renal

A. Gastrointestinal

A patient with nasogastric suctioning is experiencing diarrhea. The patient is ordered a morning dose of Lasix 20mg IV. Patient's potassium level is 3.0. What is your next nursing intervention? A. Hold the dose of Lasix and notify the doctor for further orders B. Administered the Lasix and notify the doctor for further orders C. Turn off the nasogastric suctioning and administered a laxative D. No intervention is need the potassium level is within normal range

A. Hold the dose of Lasix and notify the doctor for further orders

On admission, a patient blood alcohol limit is greater than 400 mg/dL. The patient reports drinking a 12 pack of beer on a daily basis. Which of the following conditions is this patient MOST at risk for? A. Hypomagnesemia B. Hypermagnesemia C. Hyponatremia D. Hypernatremia

A. Hypomagnesemia

A patient has a sodium level of 130. What is this condition called? A. Hyponatremia B. Hypernatremia C. Normal Sodium Level D. Hypercalcemia

A. Hyponatremia

A patient's calcium level is 6.9. Which of the following is a nursing priority? A. Initiate seizure precautions B. Educate patient about foods rich in calcium C. Administer Calcitonin D. Administer Vitamin D supplements as ordered

A. Initiate seizure precautions

Which of the following is not a symptom of hyperkalemia? A. Positive Chvostek's sign B. Decreased blood pressure C. Muscle twitches/cramps D. Weak and slow heart rate

A. Positive Chvostek's sign

A patient has a potassium level of 9.0. Which nursing intervention is priority? A. Prepare the patient for dialysis and place the patient on a cardiac monitor B. Administer Spironolactone C. Place patient on a potassium restrictive diet D. Administer a laxative

A. Prepare the patient for dialysis and place the patient on a cardiac monitor

A client with cirrhosis of the liver develops ascites. Which of the following orders would the nurse expect? A. Restrict fluid to 1000 mL per day. B. Ambulate 100 ft. three times per day. C. High-sodium diet. D. Maalox 30 ml P.O. BID.

A. Restrict fluid to 1000 mL per day.

Fill In The Blank The method by which a nurse manually expresses urine from the bladder by pressing gently on the lower abdomen is the _____ method.

ANS: Cred The Cred method calls for the manual expressing of urine from the bladder be gently pressing down on the lower abdomen and pressing the bladder.

A patient with a pituitary tumor will most likely exhibit symptoms of: A. alteration in visual acuity. B. frequent diarrhea. C. alterations in blood glucose. D. urticaria.

A. alteration in visual acuity. Because of its location in the middle of the skull, adjacent to the optic nerve and brain, pressure on this area can cause an increase in intracranial pressure (ICP). As a pituitary tumor enlarges, ICP continues to increase. Common symptoms of increasing ICP are nausea, headache, vomiting, and decreasing visual acuity.

Lithium is known to affect the parathyroid by increasing ______ levels and decreasing _____ levels? A. calcium, phosphate B. phosphate, calcium C. calcium, sodium D. sodium, calcium

A. calcium, phosphate

Muscular twitching and cramps in the muscles following thyroidectomy may indicate: A. damage to the parathyroid glands during surgery. B. early signs of epilepsy. C. damage to the cervical nerves. D. potassium depletion.

A. damage to the parathyroid glands during surgery. Tetany actually results from injury to, or accidental removal of, the parathyroid glands. PTH is important in regulating body calcium and phosphorus levels, and a deficiency of PTH produces muscle cramps, twitching of the muscles, and, in some cases, severe convulsions. These symptoms represent a medical emergency and must be reported to the physician at once.

The nurse is caring for a patient after a total thyroidectomy. The nurse's priority should be to: A. maintain the patient in semi-Fowler's position, with her head back and neck supported by pillows. B. encourage the patient to turn her head side to side, to promote drainage of oral secretions. C. maintain the patient in a supine position, with sandbags placed on either side of her head and neck. D. encourage the patient to cough and deep-breathe every 2 hours, with her neck in a flexed position.

A. maintain the patient in semi-Fowler's position, with her head back and neck supported by pillows. Following thyroidectomy, the patient should be placed in semi-Fowler's position to decrease swelling. Edema would exert pressure on the patient's airway, potentially compromising patency. Each of the other actions would increase the chances of postoperative complications, including bleeding, swelling, and airway obstruction.

Secondary obesity

can result from various congenital anomalies, chromosomal anomalies, metabolic problems, CNS lesions and disorder, or drugs.

Fill In The Blank A nurse explains that the normal bladder will empty when it reaches the capacity of _____ to _____ mL.

ANS: 200; 250 The urge to void will occur when the bladder is holding 200 to 250 mL of urine.

What is the cause of symptomatic incontinence? a. Colorectal disease b. Gastrocolic reflex c. Constipation d. Nerve damage

ANS: A Symptomatic incontinence is the result of colorectal disease. Medical care should be sought to identify and treat the cause.

A patient with a spinal cord injury has recently begun using reflex training to empty his bladder. The nurse is doing a catheterization to check for residual volume. What should the residual volume be to indicate reflex training is effective? a. Less than 100 mL b. Less than 200 mL c. Less than 400 mL d. Less than 500 mL

ANS: A Ideally, the residual volume will be less than 100 mL.

What does the uroflowmetry diagnostic tool measure? a. Voiding duration b. Specific gravity of urine c. Effectiveness of the detrusor muscle d. General bladder tone

ANS: A The uroflowmetry is a diagnostic tool designed to measure voiding duration, rate, and amount.

What should a nurse include when providing instructions to a patient as to what to do when feeling the urge to void? (Select all that apply.) a. Breathe deeply and try to relax. b. Perform several Kegel maneuvers without resting in between. c. Walk to the bathroom at a normal pace while performing Kegel maneuvers. d. Distract herself with a book or a television program. e. Stop what she is doing and sit down or stand quietly.

ANS: A, B, C, E Breathing deeply, trying to relax, and performing Kegel maneuvers are all helpful in urge suppression. Distraction is seldom effective.

What foods should a nurse explain to a patient can cause diarrhea? a. Cheese b. Cabbage c. Rice d. Yogurt

ANS: B Foods such as cabbage, raw vegetables, and spicy foods can cause diarrhea. Cheese, rice, and yogurt thicken stool.

A patient is having problems with fecal incontinence. What should the nurse encourage the patient to include in the diet to help with this problem? a. Raw fruits and vegetables b. Potatoes and bread c. Coffee and tea d. Prune and grape juice

ANS: B Foods that thicken the stool, such as potatoes, bread, bananas, rice, cheese, yogurt, oatmeal, oat bran, boiled milk, and pasta, should be encouraged.

A patient, talking to a home health nurse about urinary incontinence, gives the nurse a list of the current medications she is taking. What medication should the nurse recognize as possibly contributing to the patients urinary incontinence? a. Methylcellulose (Citrucel) b. Diazepam (Valium) c. Simvastatin (Zocor) d. Digoxin (Lanoxin)

ANS: B Valium is a sedative that can increase the incidence of incontinency of urine.

A patient tells a nurse that his bowel movements normally occur every morning after breakfast. What should the nurse understand as the rationale for this occurrence? a. Fecal overflow b. Gastrocolic reflex c. Autonomic dysreflexia d. Lack of sphincter control

ANS: B When food enters the stomach, it stimulates activity throughout the digestive tract and causes the movement of fecal mass into the rectum.

What education should a nurse provide to a patient diagnosed with anorectal incontinence? a. Take a daily laxative. b. Increase fiber in the diet. c. Perform pelvic muscle exercises. d. Administer daily enemas.

ANS: C Anorectal incontinence is associated with nerve damage that causes the muscles of the pelvic floor to be weak. Pelvic muscle exercises can help strengthen these muscles. The other choices would cause the incontinence to worsen.

Which statements by a patient would indicate an accurate understanding of cytometry? (Select all that apply.) a. Drink no fluids for 6 hours after the test. b. Report a change in my abdominal girth. c. Notify the doctor if I have difficulty voiding. d. Sleep on my stomach. e. Notify my doctor if I experience burning on urination.

ANS: C, E Voiding difficulty and burning on urination are complications that should be reported to the physician. Neither fluid intake nor sleeping positions are restricted. Abdominal girth is not significant to the postcystometry recovery.

A nurse has just received a patient who had a cystoscopy from the postanesthesia recovery unit. The nurse notices that the patients urine is tinged with pink. What is the first action the nurse should implement? a. Call the physician. b. Record the assessment in the patients record. c. Encourage the patient to drink plenty of fluids. d. Prepare the patient for a return to surgery.

ANS: C Pink-tinged urine is normal at first. Encouraging fluids will help flush the patients bladder, and then the nurse should document both the assessment and implementation.

A physicians admission report states that a patient has a history of tarry stools. What should the nurse anticipate when assessing characteristics of this patients stool? a. Brown and formed b. Bright red and liquid c. Black and sticky d. Clay colored and pasty

ANS: C Tarry is used to describe stools that are shiny, sticky, and black, which is usually an indication of blood in the stool.

A nurse is cleaning a patient with fecal incontinence when the patient says, This is so embarrassing, and it makes me really angry. What is the nurses best response? a. Dont worry about it; its my job to clean you up. b. If you would have called me sooner, this wouldnt have happened. c. Do you feel angry and embarrassed? d. Would you rather let your family clean you up?

ANS: C The nurse should use therapeutic communications of reflection to validate the patients feelings.

A patient who is scheduled for an urodynamic test asks the nurse why he is having this test. What is the nurses best response? a. To test the capacity of the bladder. b. To see how much urine is left in the bladder after you have voided. c. To test the function of the nerves and muscles of the bladder. d. To detect involuntary passage of urine.

ANS: C Urodynamic procedures assess the neuromuscular function of the lower urinary tract.

What symptom of PUD is typically described as *sharp, burning, or gnawing* pain?

dyspepsia

What is the most commonly reported symptom associated with PUD?

dyspepsia (indigestion/heartburn)

A nurse is instructing a patient on the procedure for a clean-catch urine specimen. The patient has tried several times but is having difficulty understanding the instructions. What is the best action for the nurse to implement? a. Take whatever specimen the patient can obtain. b. Provide the patient with a clean bedpan to obtain the specimen. c. Ask the laboratory personnel to come and obtain a urine specimen. d. Call the physician for a catheterization order.

ANS: D If the patient cannot cooperate with the clean-catch procedure, catheterization may be necessary.

A patient being assessed by the physician states, I wet my pants every time I cough. The nurse recognizes this as which type of incontinence? a. Reflex b. Overflow c. Urge d. Stress

ANS: D Stress incontinence is the involuntary loss of small amounts of urine during physical activity that increases abdominal pressure, such as coughing, laughing, sneezing, and lifting.

Fill IN The Blank A patient complains, My allergies make me sneeze and urinate in my pants. I take my allergy drug and I urinate in my pants even more. The nurse assesses that the drug the patient is referring to is a(n) _____.

ANS: antihistamine Many antihistamine preparations increase the incidence of incontinence.

D (Sometimes in thyroid surgery, the parathyroid glands are removed, producing a disturbance in calcium metabolism. Tetany is usually treated with IV calcium gluconate.)

Accidental removal of one or both parathyroid glands can occur during a thyroidectomy. Which of the following is used to treat tetany? a) Propylthiouracil (PTU) b) Synthroid c) Tapazole d) Calcium gluconate

A client is evaluated for severe pain in the right upper abdominal quadrant, which is accompanied by nausea and vomiting. The physician diagnoses acute cholecystitis and cholelithiasis. For this client, which nursing diagnosis takes top priority? a) Acute pain related to biliary spasms b) Imbalanced nutrition: Less than body requirements related to biliary inflammation c) Anxiety related to unknown outcome of hospitalization d) Deficient knowledge related to prevention of disease recurrence

Acute pain related to biliary spasms The chief symptom of cholecystitis is abdominal pain or biliary colic. Typically, the pain is so severe that the client is restless and changes positions frequently to find relief. Therefore, the nursing diagnosis of Acute pain related to biliary spasms takes highest priority. Until the acute pain is relieved, the client can't learn about prevention, may continue to experience anxiety, and can't address nutritional concerns

Laboratory results for a patient with a large draining wound show a serum sodium decrease from 138 mEq/L to 131 mEq/L. Which action would the nurse take first? A. Establish IV access B. Assess for orthostatic hypotension C. Assess the patient's respiratory status D. Notify the health care provider of laboratory results

C

The nurse is reviewing the prescription for a client admitted to the hospital with a diagnosis of acute pancreatitis. Which interventions would the nurse expect to be prescribed for the client? Select all that apply. 1. Administer antacids as prescribed 2. Encourage coughing and deep breathing 3. Administer anticholinergics as prescribed 4. Give small, frequent high-calorie feedings 5. Maintain the client in a supine and flat position 6. Give meperidine (Demerol) as prescribed for pain.

Ans: 1, 2, 3, 6 The client with acute pancreatitis normally is placed on NPO status to rest the pancreas and suppress gastrointestinal secretions. Because abdominal pain is a prominent symptom pancreatitis, pain medication such as meperidine is prescribed. Some clients experience lessened pain by assuming positions that flex the trunk, with the knees drawn up to the chest. A side-lying position with the head elevated 45 degrees decreases tension on the abdomen and may help ease the pain. The client is susceptible to respiratory infections because the retroperitoneal fluid raises the diaphragm, which causes the client to take shallow, guarded abdominal breaths. Therefore measures such as turning, coughing, and deep breathing are instituted. Antacids and anticholinergics may be prescribed to suppress gastrointestinal secretions.

A client has been admitted to the hospital with a diagnosis of acute pancreatitis and the nurse is assessing the client's pain. What type of pain is consistent with this diagnosis? 1. Burning and aching, located in the left lower quadrant and radiating to the hip 2. Severe and unrelenting, located in the epigastric area and radiating to the back 3. Burning and aching, located in the epigastric are and radiating to the umbilicus 4. Severe and unrelenting, located in the left lower quadrant and radiating to the groin

Ans: 2 The pain associated with acute pancreatitis is often severe and unrelenting, is located in the epigastric region, and radiates to the back. The other options are incorrect.

The nurse will be teaching self-management to patients after gastric bypass surgery. Which information will the nurse plan to include? • Drink fluids between meals but not with meals. • Choose high-fat foods for at least 30% of intake. • Developing flabby skin can be prevented by exercise. • Choose foods high in fiber to promote bowel function.

Ans: A Feedback: Intake of fluids with meals tends to cause dumping syndrome and diarrhea. Food choices should be low in fat and fiber. Exercise does not prevent the development of flabby skin.

When teaching a patient about testing to diagnose metabolic syndrome, which topic would the nurse include? • Blood glucose test • Cardiac enzyme tests • Postural blood pressures • Resting electrocardiogram

Ans: A Feedback: A fasting blood glucose test >100 mg/dL is one of the diagnostic criteria for metabolic syndrome. The other tests are not used to diagnose metabolic syndrome although they may be used to check for cardiovascular complications of the disorder.

The nurse is coaching a community group for individuals who are overweight. Which participant behavior is an example of the best exercise plan for weight loss? • Walking for 40 minutes 6 or 7 days/week • Lifting weights with friends 3 times/week • Playing soccer for an hour on the weekend • Running for 10 to 15 minutes 3 times/week

Ans: A Feedback: Exercise should be done daily for 30 minutes to an hour. Exercising in highly aerobic activities for short bursts or only once a week is not helpful and may be dangerous in an individual who has not been exercising. Running may be appropriate, but a patient should start with an exercise that is less stressful and can be done for a longer period. Weight lifting is not as helpful as aerobic exercise in weight loss.

A client is in the bariatric clinic 1 month after having gastric bypass surgery. The client is crying and says I didnt know it would be this hard to live like this. What response by the nurse is best? A. Assess the clients coping and support systems. B. Inform the client that things will get easier. C. Re-educate the client on needed dietary changes. D. Tell the client lifestyle changes are always hard.

Ans: A Feedback: The nurse should assess this clients coping styles and support systems in order to provide holistic care. The other options do not address the clients distress.

A 40-year-old obese woman reports that she wants to lose weight. Which question should the nurse ask first? • What factors led to your obesity? • Which types of food do you like best? • How long have you been overweight? • What kind of activities do you enjoy?

Ans: A Feedback: The nurse should obtain information about the patients perceptions of the reasons for the obesity to develop a plan individualized to the patient. The other information also will be obtained from the patient, but the patient is more likely to make changes when the patients beliefs are considered in planning.

A client has been prescribed lorcaserin (Belviq). What teaching is most appropriate? A. Increase the fiber and water in your diet. B. Reduce fat to less than 30% each day. C. Report dry mouth and decreased sweating. D. Lorcaserin may cause loose stools for a few days.

Ans: A Feedback: This drug can cause constipation, so the client should increase fiber and water in the diet to prevent this from occurring. Reducing fat in the diet is important with orlistat. Lorcaserin can cause dry mouth but not decreased sweating. Loose stools are common with orlistat.

Which electrolyte laboratory values indicate to the nurse monitoring a client with adrenal insufficiency undergoing IV therapy with hydrocortisone that the client is responding positively to this drug therapy? A. serum sodium 147 mEq/L; serum potassium 7.1 mEq/L B. serum sodium 137 mEq/L; serum potassium 4.9 mEq/L C. serum sodium 127 mEq/L; serum potassium 2.8 mEq/L D. serum sodium 119 mEq/L; serum potassium 6.2 mEq/L

B

Which finding for a patient who has been taking orlistat (Xenical) is most important to report to the health care provider? • The patient frequently has liquid stools. • The patient is pale and has many bruises. • The patient complains of bloating after meals. • The patient is experiencing a weight loss plateau.

Ans: B Feedback: Because orlistat blocks the absorption of fat-soluble vitamins, the patient may not be receiving an adequate amount of vitamin K, resulting in a decrease in clotting factors. Abdominal bloating and liquid stools are common side effects of orlistat and indicate that the nurse should remind the patient that fat in the diet may increase these side effects. Weight loss plateaus are normal during weight reduction.

Which nursing action is appropriate when coaching obese adults enrolled in a behavior modification program? • Having the adults write down the caloric intake of each meal • Asking the adults about situations that tend to increase appetite • Suggesting that the adults plan rewards, such as sugarless candy, for achieving their goals • Encouraging the adults to eat small amounts frequently rather than having scheduled meals

Ans: B Feedback: Behavior modification programs focus on how and when the person eats and de-emphasize aspects such as calorie counting. Nonfood rewards are recommended for achievement of weight-loss goals. Patients are often taught to restrict eating to designated meals when using behavior modification.

Which information will the nurse prioritize in planning preoperative teaching for a patient undergoing a Roux-en-Y gastric bypass? • Educating the patient about the nasogastric (NG) tube • Instructing the patient on coughing and breathing techniques • Discussing necessary postoperative modifications in lifestyle • Demonstrating passive range-of-motion exercises for the legs

Ans: B Feedback: Coughing and deep breathing can prevent major postoperative complications such as carbon monoxide retention and hypoxemia. Information about passive range of motion, the NG tube, and postoperative modifications in lifestyle will also be discussed, but avoidance of respiratory complications is the priority goal after surgery.

Which assessment action will help the nurse determine if an obese patient has metabolic syndrome? A. Take the patients apical pulse. B. Check the patients blood pressure. C. Ask the patient about dietary intake. D. Dipstick the patients urine for protein.

Ans: B Feedback: Elevated blood pressure is one of the characteristics of metabolic syndrome. The other information also may be obtained by the nurse, but it will not assist with the diagnosis of metabolic syndrome.

A client is awaiting bariatric surgery in the morning. What action by the nurse is most important? • Answering questions the client has about surgery • Beginning venous thromboembolism prophylaxis • Informing the client that he or she will be out of bed tomorrow • Teaching the client about needed dietary changes

Ans: B Feedback: Morbidly obese clients are at high risk of venous thromboembolism and should be started on a regimen to prevent this from occurring as a priority. Answering questions about the surgery is done by the surgeon. Teaching is important, but safety comes first.

Which adult will the nurse plan to teach about risks associated with obesity? • Man who has a BMI of 18 kg/m2 • Man with a 42 in waist and 44 in hips • Woman who has a body mass index (BMI) of 24 kg/m2 • Woman with a waist circumference of 34 inches (86 cm)

Ans: B Feedback: The waist-to-hip ratio for this patient is 0.95, which exceeds the recommended level of <0.80. A patient with a BMI of 18 kg/m2 is considered underweight. A BMI of 24 kg/m2 is normal. Health risks associated with obesity increase in women with a waist circumference larger than 35 in (89 cm) and men with a waist circumference larger than 40 in (102 cm).

After the nurse teaches a patient about the recommended amounts of foods from animal and plant sources, which menu selections indicate that the initial instructions about diet have been understood? • 3 oz of lean beef, 2 oz of low-fat cheese, and a tomato slice • 3 oz of roasted pork, a cup of corn, and a cup of carrot sticks • Cup of tossed salad and nonfat dressing topped with a chicken breast • Half cup of tuna mixed with nonfat mayonnaise and a half cup of celery

Ans: B Feedback: This selection is most consistent with the recommendation of the American Institute for Cancer Research that one third of the diet should be from animal sources and two thirds from plant source foods. The other choices all have higher ratios of animal origin foods to plant source foods than would be recommended.

After bariatric surgery, a patient who is being discharged tells the nurse, I prefer to be independent. I am not interested in any support groups. Which response by the nurse is best? • I hope you change your mind so that I can suggest a group for you. • Tell me what types of resources you think you might use after this surgery. • Support groups have been found to lead to more successful weight loss after surgery. • Because there are many lifestyle changes after surgery, we recommend support groups.

Ans: B Feedback: This statement allows the nurse to assess the individual patients potential needs and preferences. The other statements offer the patient more information about the benefits of support groups, but fail to acknowledge the patients preferences.

A client just returned to the surgical unit after a gastric bypass. What action by the nurse is the priority? • Assess the clients pain. • Check the surgical incision. • Ensure an adequate airway. • Program the morphine pump.

Ans: C Feedback: All actions are appropriate care measures for this client; however, airway is always the priority. Bariatric clients tend to have short, thick necks that complicate airway management.

1. Which statement by the nurse is most likely to help a morbidly obese 22-year-old man in losing weight on a 1000-calorie diet? • It will be necessary to change lifestyle habits permanently to maintain weight loss. • You will decrease your risk for future health problems such as diabetes by losing weight now. • You are likely to notice changes in how you feel with just a few weeks of diet and exercise. • Most of the weight that you lose during the first weeks of dieting is water weight rather than fat.

Ans: C Feedback: Motivation is a key factor in successful weight loss and a short-term outcome provides a higher motivation. A 22-year-old patient is unlikely to be motivated by future health problems. Telling a patient that the initial weight loss is water will be discouraging, although this may be correct. Changing lifestyle habits is necessary, but this process occurs over time and discussing this is not likely to motivate the patient.

Positive Trousseau and Chvostek signs are consistent with which electrolyte imbalance? A. Hypokalemia B. Hyperkalemia C. Hypocalcemia D. Hypercalcemia

C

A few months after bariatric surgery, a 56-year-old man tells the nurse, My skin is hanging in folds. I think I need cosmetic surgery. Which response by the nurse is most appropriate? A. The important thing is that you are improving your health. B. The skinfolds will disappear once most of the weight is lost. C. Cosmetic surgery is a possibility once your weight has stabilized. D. Perhaps you would like to talk to a counselor about your body image.

Ans: C Feedback: Reconstructive surgery may be used to eliminate excess skinfolds after at least a year has passed since the surgery. Skinfolds may not disappear over time, especially in older patients. The response, The important thing is that your weight loss is improving your health, ignores the patients concerns about appearance and implies that the nurse knows what is important. Whereas it may be helpful for the patient to talk to a counselor, it is more likely to be helpful to know that cosmetic surgery is available.

After successfully losing 1 lb weekly for several months, a patient at the clinic has not lost any weight for the last month. The nurse should first • review the diet and exercise guidelines with the patient. • instruct the patient to weigh and record weights weekly. • ask the patient whether there have been any changes in exercise or diet patterns. • discuss the possibility that the patient has reached a temporary weight loss plateau.

Ans: C Feedback: The initial nursing action should be assessment of any reason for the change in weight loss. The other actions may be needed, but further assessment is required before any interventions are planned or implemented.

What information will the nurse include for an overweight 35-year-old woman who is starting a weightloss plan? • Weigh yourself at the same time every morning and evening. • Stick to a 600- to 800-calorie diet for the most rapid weight loss. • Low carbohydrate diets lead to rapid weight loss but are difficult to maintain. • Weighing all foods on a scale is necessary to choose appropriate portion sizes.

Ans: C Feedback: The restrictive nature of fad diets makes the weight loss achieved by the patient more difficult to maintain. Portion size can be estimated in other ways besides weighing. Severely calorie-restricted diets are not necessary for patients in the overweight category of obesity and need to be closely supervised. Patients should weigh weekly rather than daily.

A 61-year-old man is being admitted for bariatric surgery. Which nursing action can the nurse delegate to unlicensed assistive personnel (UAP)? • Demonstrate use of the incentive spirometer. • Plan methods for bathing and turning the patient. • Assist with IV insertion by holding adipose tissue out of the way. • Develop strategies to provide privacy and decrease embarrassment.

Ans: C Feedback: UAP can assist with IV placement by assisting with patient positioning or holding skinfolds aside. Planning for care and patient teaching require registered nurse (RN)level education and scope of practice.

The nurse is caring for a 54-year-old female patient on the first postoperative day after a Roux-en-Y gastric bypass procedure. Which assessment finding should be reported immediately to the surgeon? • Bilateral crackles audible at both lung bases • Redness, irritation, and skin breakdown in skinfolds • Emesis of bile-colored fluid past the nasogastric (NG) tube • Use of patient-controlled analgesia (PCA) several times an hour for pain

Ans: C Feedback: Vomiting with an NG tube in place indicates that the NG tube needs to be repositioned by the surgeon to avoid putting stress on the gastric sutures. The nurse should implement actions to decrease skin irritation and have the patient cough and deep breathe, but these do not indicate a need for rapid notification of the surgeon. Frequent PCA use after bariatric surgery is expected.

After vertical banded gastroplasty, a 42-year-old male patient returns to the surgical nursing unit with a nasogastric tube to low, intermittent suction and a patient-controlled analgesia (PCA) machine for pain control. Which nursing action should be included in the postoperative plan of care? A. Offer sips of fruit juices at frequent intervals. B. Irrigate the nasogastric (NG) tube frequently. C. Remind the patient that PCA use may slow the return of bowel function. D. Support the surgical incision during patient coughing and turning in bed.

Ans: D Feedback: The incision should be protected from strain to decrease the risk for wound dehiscence. The patient should be encouraged to use the PCA because pain control will improve the cough effort and patient mobility. NG irrigation may damage the suture line or overfill the stomach pouch. Sugar-free clear liquids are offered during the immediate postoperative time to decrease the risk for dumping syndrome.

The nurse is instructing a patient with chronic pancreatitis on measures to prevent further attacks. What information should be provided (select all that apply)? a. Avoid nicotine b. Eat bland foods c. Observe stools for steatorrhea d. Eat high-fat, low-protein, high-carbohydrate meals e. Take prescribed pancreatic enzymes immediately following meals.

Ans: a, b, c Measures to prevent attacks of pancreatitis are those that decrease the stimulation of the pancreas. Lower fat intake and foods that are less stimulating and irritating (bland) should be encouraged. Higher carbohydrates are less stimulating. Avoid alcohol and nicotine, since both stimulate the pancreas. Monitor for steatorrhea to determine effectiveness of the enzymes and because it may indicate worsening pancreatic function. Pancreatic enzymes should be taken with, not after, meals.

Nursing management of the patient with acute pancreatitis includes (select all that apply) a. checking for signs of hypocalcemia. b. providing a diet low in carbohydrates. c. giving insulin based on a sliding scale. d. observing stools for signs of steatorrhea. e. monitoring for infection, particularly respiratory tract infection.

Ans: a, e

Combined with clinical manifestations, what is the laboratory finding that is most commonly used to diagnose acute pancreatitis? a. Increased serum calcium b. Increased serum amylase c. Increased urinary amylase d. Decreased serum glucose

Ans: b Although serum lipase levels and urinary amylasy levels are increased, an increased serum amylase level is the criterion most commonly used to diagnose acute pancreatitis in the first 24 to 72 hours. Serum calcium levels are decreased.

What is the patient with chronic pancreatitis more likely to have than the patient with acute pancreatitis? a. The need to abstain from alcohol b. Experience acute abdominal pain c. Malabsorption and diabetes mellitus d. Require a high-carbohydrate, high-protein, low-fat diet

Ans: c Chronic damage to the pancreas causes a deficiency of digestive enzymes and insulin resulting in malabsorption and diabetes mellitus. Abstinence from alcohol is necessary in both types of pancreatitis, as is a high-carbohydrate, high protein, and low-fat diet. Although abdominal pain is a major manifestation of chronic pancreatitis, more commonly a constant heavy, gnawing feeling occurs.

What treatment measure is used in the management of the patient with acute pancreatitis? a. Surgery to remove the inflamed pancreas b. Pancreatic enzyme supplements administered with meals c. Nasogastric (NG) suction to prevent gastric contents from entering the duodenum d. Endoscopic pancreatic sphincterotomy using endoscopic retrograde cholangiopancreatography (ERCP)

Ans: c Pancreatic rest and suppression of secretions are promoted by preventing any gastric contents from entering the duodenum, which would stimulate pancreatic activity. Surgery is not indicated for acute pancreatitis but may be used to drain abscesses or cysts. An endoscopic retrograde cholangiopancreatography (ERCP) pancreatic sphincterotomy may be performed when pancreatitis is related to gallstones. Pancreatic enzyme supplements are necessary in chronic pancreatitis if a deficiency in secretion occurs.

A patient with acute pancreatitis has a nursing diagnosis of pain related to distention of the pancreas and peritoneal irritation. In addition to effective use of analgesics, what should the nurse include in this patient's plan of care? a. Provide diversional activities to distract the patient from the pain. b. Provide small, frequent meals to increase the patient's tolerance to food. c. Position the patient on the side with the head of the bed elevated 45 degrees of pain relief. d. Ambulate the patient every 3 to 4 hours to increase circulation and decrease abdominal congestion.

Ans: c Positions that flex the trunk and draw the knees up to the abdomen help to relieve the pain of acute pancreatitis and positioning of the patient on the side with the head elevated decreases abdominal tension. Diversional techniques are not as helpful as positioning in controlling the pain. The patient is usually NPO because food intake increases the pain and inflammation. Bed rest is indicated during the acute attack because of hypovolemia and pain.

The nurse determines that further discharge instruction is needed when the patient with acute pancreatitis makes which statement? a. "I should observe for fat in my stools." b. "I must not use alcohol to prevent future attacks of pancreatitis." c. "I shouldn't eat any salty foods or foods with high amounts of sodium." d. "I will need to continue to monitor my blood glucose levels until my pancreas is healed."

Ans: c Sodium restriction is not indicated for patients recovering from acute pancreatitis but the stools should be observed for steatorrhea, indicating that fat digestion is impaired, and glucose levels may be monitored for indication of impaired B-cell function. Alcohol is a primary cause of pancreatitis and should not be used.

Which complication of acute pancreatitis requires prompt surgical drainage to prevent sepsis? a. Tetany b. Pseudocyst c. Plerual effusion d. Pancreatic abscess

Ans: d A pancreatic abscess is a collection of pus that must be drained to prevent infection of adjacent organs and sepsis. Tetany from hupocalcemia is treated with IV calcium gluconate (10%). Although pseudocysts usually resolve spontaneously, they may be treated with surgical, percutaneous catheter, or endoscopic drainage to prevent perforation. Pleural effusion is treated by treating the cause (pancreatitits) and monitoring for respiratory distress and oxygen saturation.

When assessing a patient with acute pancreatitis, the nurse would expect to find: a. hyperactive bowel sounds. b. hypertension and tachycardia. c. a temperature greater than 102 F (38.9 C). d. Severe midepigastric or left upper quadrant (LUQ) pain.

Ans: d the predominant symptom of acute pancreatitis is severe, deep abdominal pain that is usually located in the left upper quadrant (LUQ) but may be in the midepigastrium. Bowel sounds are decreased or absent, temperature is elevated only slightly, and the patient has hypovolemia and may manifest symptoms of shock.

The nurse is planning care for a patient following an incisional cholecystectomy for cholelithiasis. Which of the following interventions is the highest nursing priority for this patient? a) Performing range-of-motion (ROM) leg exercises hourly while the patient is awake b) Teaching the patient to choose low-fat foods from the menu c) Assisting the patient to turn, cough, and deep breathe every 2 hours d) Assisting the patient to ambulate the evening of the operative day

Assisting the patient to turn, cough, and deep breathe every 2 hours Assessment should focus on the patient's respiratory status. If a traditional surgical approach is planned, the high abdominal incision required during surgery may interfere with full respiratory excursion. The other nursing actions are also important, but are not as high a priority as ensuring adequate ventilation.

A nurse is caring for a client who has a nasogastric tube attached to low intermittent suctioning. The nurse should monitor for which of the following electrolyte imbalance? A. Hypercalcemia B. Hyponatremia C. Hyperphosphatemia D. Hyperkalemia

B

A patient who recently experienced an anterior neck injury reports frequent and painful muscle spasms in the calf during sleep. Which condition would the nurse suspect? A. Hypokalemia B. Hypocalcemia C. Hyponatremia D. Hypophosphatemia

B

What is the primary treatment approach when dealing with chronic gastritis?

eliminating causative factors (H.pylori)

The nurse is planning the care of a patient with hyperthyroidism. What should the nurse specify in the patients meal plan? A) A clear liquid diet, high in nutrients B) Small, frequent meals, high in protein and calories C) Three large, bland meals a day D) A diet high in fiber and plant-sourced fat

B (A patient with hyperthyroidism has an increased appetite. The patient should be counseled to consume several small, well-balanced meals. High-calorie, high-protein foods are encouraged. A clear liquid diet would not satisfy the patients caloric or hunger needs. A diet rich in fiber and fat should be avoided because these foods may lead to GI upset or increase peristalsis.)

The physician has ordered a fluid deprivation test for a patient suspected of having diabetes insipidus. During the test, the nurse should prioritize what assessments? A) Temperature and oxygen saturation B) Heart rate and BP C) Breath sounds and bowel sounds D) Color, warmth, movement, and sensation of extremities

B (The fluid deprivation test is carried out by withholding fluids for 8 to 12 hours or until 3% to 5% of the body weight is lost. The patients condition needs to be monitored frequently during the test, and the test is terminated if tachycardia, excessive weight loss, or hypotension develops. Consequently, BP and heart rate monitoring are priorities over the other listed assessments.)

A patient with a diagnosis of syndrome of inappropriate antidiuretic hormone secretion (SIADH) is being cared for on the critical care unit. The priority nursing diagnosis for a patient with this condition is what? A) Risk for peripheral neurovascular dysfunction B) Excess fluid volume C) Hypothermia D) Ineffective airway clearance

B (The priority nursing diagnosis for a patient with SIADH is excess fluid volume, as the patient retains fluids and develops a sodium deficiency. Restricting fluid intake is a typical intervention for managing this syndrome. Temperature imbalances are not associated with SIADH. The patient is not at risk for neurovascular dysfunction or a compromised airway.)

once bleeding is controlled, patient should undergo what

endoscope -- to see where the bleed is coming from

After and EGD, how often are VS assessed?

every 30 min until sedation wears off

3. The nurse is developing a teaching plan for a client with stress incontinence. Which of the following instructions should be included? A) Avoid activities that are stressful and upsetting. B) Avoid caffeine and alcohol. C) Do not wear a girdle. D)Limit physical exertion.

B) Avoid caffeine and alcohol.

A hypertensive patient was brought to the emergency department with a hear rate of 115 beats/min and an abnormal ECG showing shortened QT interval. The laboratory findings show a serum calcium level of 11mg/dL. Which nursing intervention would help stabilize the patient, select all that apply letter only. A. Administering thiazide diuretics B. Administering high-ceiling or loop diuretics C. Administering 0.9% normal saline IV D. Administering NSAIDs E. Administering lactated Ringer's solution IV

B, C

A 77-year-old women who has CHF is brought to the emergency department after she has had diarrhea for 3 days. The family tells the nurse that she has not been eating or drinking well but that she has been taking her diuretics and other medications. Her laboratory results include a potassium level of 3.0 mEq/L. Which of the information would the nurse include in the patient's medication teaching? Select all that apply, letter only. A. Diuretics' increase fluid retention B. Laxatives can lead to fluid imbalance C. It is important to weigh daily at the same time D. Diuretics can lead to fluid and electrolyte imbalances. E. Daily weights are a poor indicator of fluid loss or gain.

B, C, D

Primary obesity

excess calorie intake over energy expenditure for the body's metabolic demands

A patient with hypovolemic hyponatremia is started on IV fluids. Which of the following fluids do you expect the patient to be started on? A. 0.45% Saline B. 3% Saline C. D5W D. 0.33% Saline

B. 3% Saline

A patient with hypovolemic hyponatremia is started on IV fluids. Which of the following fluids do you expect the patient to be started on?* A. 0.45% Saline B. 3% Saline C. D5W D. 0.33% Saline

B. 3% Saline

A postoperative patient has the following ABG results: pH 7.30; PaCO2 60 mm Hg, PaO2 80 mm Hg; Bicarbonate 24 mEq/L and o2 sat at 96%. Which action would the nurse take? A. Administer ox by nasal cannula B. Inform the charge urse that no changes in therapy are needed C. Encourage the patient to do deep breathing, and assist with repositioning D. Request a prescription for sodium bicarbonate from the health care provider

C

A patient's potassium level is 3.0. Which foods would you encourage the patient to consume? A. Cheese, collard greens, and fish B. Avocados, strawberries, and potatoes C. Tofu, oatmeal, and peas D. Raisins, yogurt, and cauliflower

B. Avocados, strawberries, and potatoes

An EKG shows a shortened QT interval. Which lab value below would be indicative of this change? A. Calcium level of 8.0 B. Calcium level of 12.0 C. Calcium level of 8.7 D Calcium level of 9.2

B. Calcium level of 12.0

Stimulation of the facial nerve via the masseter muscle causes twitching of the nose/lips in hypocalcemia is known as? A. Trousseau's Sign B. Chvostek's Sign C. Homan's Sign D. Goodell's Sign

B. Chvostek's Sign

A patient has a sodium level of 123 and presents with confusion. The doctor diagnoses the patient with Syndrome of Inappropriate Antidiuretic Hormone Secretion (SIADH). Which type of hyponatremia is this? A. Hypovolemic B. Euvolemic C. Hypervolemic D. Antivolemic

B. Euvolemic

obesity

excessively high amount off body fat or adipose tissue

A patient with Celiac disease is at risk for which of the following? A. Hypokalemia B. Hypocalcemia C. Hypomagnesemia D. Hypercalcemia

B. Hypocalcemia

A patient with Addison's disease will most likely exhibit which symptom? A. Bradycardia B. Hypotension C. Hirsutism D. Purple striae

B. Hypotension Addison's disease is characterized by decreased function of the adrenal cortex, resulting in a deficit of all three hormones secreted by the adrenal cortex. The major problems are related to insufficiencies of the mineralocorticoids and the glucocorticoids. The patient experiences generalized malaise and muscle weakness, muscle pain, orthostatic hypotension, and vulnerability to cardiac dysrhythmias.

A patient has a potassium level of 2.0. What would you expect to be ordered for this patient? A. Potassium 30 meq IV push B. Infusion of Potassium intravenously C. An oral supplement of potassium D. Intramuscular injection of Potassium

B. Infusion of Potassium intravenously

A patient with a sodium level of 112 is taking Lithium. Which of the following is a nursing priority? A. Hold further doses of Lithium B. Monitor Lithium drug level due to risk of toxicity C. Monitor potassium level due to increased risk of toxicity D. No priority is need. 112 is a normal sodium level

B. Monitor Lithium drug level due to risk of toxicity

A patient's magnesium level is 0.9. The doctor orders Magnesium Sulfate IV. Which nursing intervention takes PRIORITY? A. Assessing for hypertension B. Monitoring deep tendon reflexes C. Monitoring potassium levels D. Monitoring skin turgor

B. Monitoring deep tendon reflexes

In reviewing the electrolytes of a client, the nurse notes the serum potassium level has increased from 4.6 mEq/L to 6.1 mEq/L. Which assessment does the nurse perform first to prevent harm? A. Deep tendon reflexes B. Oxygen saturation C. Pulse rate and rhythm D. Respiratory rate and depth

C

>40

extreme obesity

In the postoperative period, the LPN/LVN should observe a patient who has had a thyroidectomy for evidence of thyroid crisis. Two common signs of thyroid crisis are: A. twitching of muscles and severe convulsions. B. extreme temperature elevation and rapid pulse rate. C. respiratory embarrassment and hoarseness. D. depression and fatigue.

B. extreme temperature elevation and rapid pulse rate. Thyroid storm (TS), also known as thyroid crisis or thyrotoxicosis, is another complication following a thyroidectomy. In the postoperative setting, the condition is caused by a sudden increase in the output of thyroxine caused by manipulation of the thyroid as it is being removed. Another cause of TS may be improper reduction of thyroid secretions before surgery. The symptoms of TS are produced by a sudden and extreme elevation of all body processes. The temperature may rise to 106° F (41.1° C) or more, the pulse increases to as much as 200 beats per minute, respirations become rapid, and the patient exhibits marked apprehension and restlessness. Unless the condition is relieved, the patient quickly passes from delirium to coma to death from heart failure.

A 42-year-old patient complains that she has been losing a lot of her hair, is fatigued, and is cold all the time. The LPN/LVN recognizes that these symptoms are indicative of: A. hyperthyroidism. B. hypothyroidism. C. hyperparathyroidism. D. hypoparathyroidism.

B. hypothyroidism Hypothyroidism can be caused by inflammation of the thyroid gland (thyroiditis) or by treatment of hyperthyroidism that results in destroying too many thyroid cells and a resultant deficit of thyroid hormone.

A patient with Cushing's disease should be instructed to: A. avoid alcoholic beverages. B. limit dietary sodium. C. increase servings of dark green leafy vegetables. D. limit the amount of dietary protein.

B. limit dietary sodium. Cushing's disease involves hyperadrenocortical secretion and a patient with Cushing's will retain sodium and water. Alcohol in moderation is allowed. There is no need to increase consumption of green leafy vegetables or protein.

Obesity Assessment Methods

BMI WHR body shape

A patient with acute pancreatitis has been started on total parenteral nutrition (TPN). Following the administration of the TPN, which of the following should the nurse plan to monitor? a) Auscultate the abdomen for bowel sounds every 4 hours b) Measure the abdominal girth every shift c) Blood glucose levels every 4 to 6 hours d) Complaints of nausea and vomiting

Blood glucose levels every 4 to 6 hours Enteral or parenteral nutrition may be prescribed. In addition to administering enteral or parenteral nutrition, the nurse monitors serum glucose levels every 4 to 6 hours.

A client with severe diarrhea reports tingling lips and foot cramps. What is the nurse's best first action to prevent harm? A. Hold the next does of the prescribed antidiarrheal drug B. Assess bowel sounds in all four abdominal quadrants C. Assess the client's response to the Chvostek test D. Increase the IV flow rate of the normal saline infusion.

C

A laboratory report for a patient shows the following results: pH 7.32; bicarbonate 24 mEq/L; partial pressure of arterial oxygen 77 mm Hg; and partial pressure of arterial carbon dioxide 48 mm Hg. These findings are consistent with which acid-base imbalance? A. Metabolic acidosis B. Metabolic alkalosis C. Respiratory acidosis D. Respiratory alkalosis

C

A nurse is assessing a client for Chvostek's sign. Which of the following techniques should the nurse use to perform the test? A. Apply a blood pressure cuff to the client's arm B. Place the stethoscope bell over the client's carotid artery C. Tap lightly on the clients cheek D. Ask the client to lower their chin to their chest

C

A nurse is caring for a client admitted with confusion and lethargy. The client was found at home unresponsive with an empty bottle of aspirin lying next to the bed. Vital signs reveal blood pressure 104/72 mm Hg, heart rate 116/min with regular rhythm, and respiratory rate 42/min and deep. Which of the following arterial blood gas findings should the nurse expect? A. pH: 7.68 pao2 (Pulse ox): 96 mm Hg paco2: 38 mm Hg hco3: 23 mEq/L B. ph: 7.48 pao2: 100 mm hg paco2: 28 mm hg hco3: 23 mEq/l C. ph: 6.98 pao2: 100 mm hg paco2: 30 mm hg hco3: 18 mEq/l D. ph: 7.58 pao2: 96 mm hg paco2 :38 mm hg hco3: 29 mEq/l

C

A patient is receiving insulin and glucose infusion therapy for hyperkalemia now has a serum potassium level of 3.6 mEq/L. Which action would the nurse take? A. Stop the infusion immediately. B. Continue the infusion at the prescribed rate C. Assess the patient's heart rate, rhythm, and respiratory status D. Slow the infusion, and increase the frequency of the vital sign assessment.

C

The handgrasp strength of a client with metabolic acidosis has diminished since the previous assessment 1 hour ago. What is the nurse's best first action? A. Measure the client's pulse and blood pressure B. Apply humidified oxygen by nasal cannula C. Assess the client's oxygen saturation D. Notify the rapid response team

C

The nurse is reviewing serum electrolytes and blood chemistry for a newly admitted patient. Which result is of concern? A. Glucose: 97 mb/dL B. Sodium 145 mEq/L C Potassium 5.9 mEq/L D. Magnesium 2.1 mEq/L

C

Which ECG finding is consistent with hyperkalemia? A. Absent T waves B. Elevated P waves C. Prolonged PR intervals D. Shortened QRS complexes

C

Which assessment parameter is useful for identifying magnesium toxicity during IV magnesium administration? A. Measuring urine output B. Monitoring serum calcium levels C. Checking deep tendon reflexes D. Asking the patient about mood changes

C

Which nursing action for a patient with hypocalcemia is appropriate to delegate to assistive personnel? A. Evaluating the patient's laboratory results B. Implementing seizure precautions for the patient C. Transferring the patient from the bed to a stretcher using a lift sheet D. collaborating with the dietitian to prescribe calcium-rich foods for this patient

C

Which nursing action is the greatest priority in the plan of care for a patient recovering from an acute episode of chronic obstructive pulmonary disease (COPD)? A. Assess nail beds every 2 hours for cyanosis B. Assess orientation every 4 hours C. Monitor the respiratory rate and effort hourly D. Monitor ABG's results as ordered

C

Which patient is at high risk for hypernatremia? A. 30-year-old on a low-salt diet B. 42-year-old receiving hypotonic fluids C. 54-year-old who is sweating profusely D. 17-year-old with a serum blood glucose level of 189 mg/dl

C

Which two electrolyte imbalances does the nurse suspect based on the following assessment data of a patient? -Hyperactive deep tendon reflexes -Numbness and tingling in hands and feet -Painful muscles contractions -Peripheral pulse is weak,thready -Reports feeling depressed -Confusion -Egg allergy -Lactose intolerance -Vegan diet A. Hypercalcemia and hypernatremia B. Hypokalemia and hypermagnesemia C. Hypomagnesemia and hypocalcemia D. Hyperphosphatemia and hypercalcemia

C

Which urine characteristics indicate to the nurse that the client being managed for diabetes insipidus is responding appropriately to interventions? A. Urine output volume increased; urine specific gravity increased b. Urine output volume increased; urine specific gravity decreased C. Urine output volume decreased; urine specific gravity increased D. Urine output volume decreased; urine specific gravity decreased.

C

The nurse is assessing a patient diagnosed with Graves disease. What physical characteristics of Graves disease would the nurse expect to find? A) Hair loss B) Moon face C) Bulging eyes D) Fatigue

C (Clinical manifestations of the endocrine disorder Graves disease include exophthalmos (bulging eyes) and fine tremor in the hands. Graves disease is not associated with hair loss, a moon face, or fatigue.)

A patient has returned to the floor after having a thyroidectomy for thyroid cancer. The nurse knows that sometimes during thyroid surgery the parathyroid glands can be injured or removed. What laboratory finding may be an early indication of parathyroid gland injury or removal? A) Hyponatremia B) Hypophosphatemia C) Hypocalcemia D) Hypokalemia

C (Injury or removal of the parathyroid glands may produce a disturbance in calcium metabolism and result in a decline of calcium levels (hypocalcemia). As the blood calcium levels fall, hyperirritability of the nerves occurs, with spasms of the hands and feet and muscle twitching. This group of symptoms is known as tetany and must be reported to the physician immediately, because laryngospasm may occur and obstruct the airway. Hypophosphatemia, hyponatremia, and hypokalemia are not expected responses to parathyroid injury or removal. In fact, parathyroid removal or injury that results in hypocalcemia may lead to hyperphosphatemia.)

The nurse is caring for a patient with a diagnosis of Addisons disease. What sign or symptom is most closely associated with this health problem? A) Truncal obesity B) Hypertension C) Muscle weakness D) Moon face

C (Patients with Addisons disease demonstrate muscular weakness, anorexia, gastrointestinal symptoms, fatigue, emaciation, dark pigmentation of the skin, and hypotension. Patients with Cushing syndrome demonstrate truncal obesity, moon face, acne, abdominal striae, and hypertension.)

A nurse caring for a patient with diabetes insipidus is reviewing laboratory results. What is an expected urinalysis finding? A) Glucose in the urine B) Albumin in the urine C) Highly dilute urine D) Leukocytes in the urine

C (Patients with diabetes insipidus produce an enormous daily output of very dilute, water-like urine with a specific gravity of 1.001 to 1.005. The urine contains no abnormal substances such as glucose or albumin. Leukocytes in the urine are not related to the condition of diabetes insipidus, but would indicate a urinary tract infection, if present in the urine.)

The nurse is teaching a patient that the body needs iodine for the thyroid to function. What food would be the best source of iodine for the body? A) Eggs B) Shellfish C) Table salt D) Red meat

C (The major use of iodine in the body is by the thyroid. Iodized table salt is the best source of iodine.)

A patient is prescribed corticosteroid therapy. What would be priority information for the nurse to give the patient who is prescribed long-term corticosteroid therapy? A) The patients diet should be low protein with ample fat. B) The patient may experience short-term changes in cognition. C) The patient is at an increased risk for developing infection. D) The patient is at a decreased risk for development of thrombophlebitis and thromboembolism.

C (The patient is at increased risk of infection and masking of signs of infection. The cardiovascular effects of corticosteroid therapy may result in development of thrombophlebitis or thromboembolism. Diet should be high in protein with limited fat. Changes in appearance usually disappear when therapy is no longer necessary. Cognitive changes are not common adverse effects.)

over 40

extreme obesity BMI

4. A client has urge incontinence. When obtaining the health history, the nurse should ask if the client has: A) Inability to empty the bladder. B) Loss of urine when coughing. C) Involuntary urination with minimal warning. D) Frequent dribbling of urine.

C) Involuntary urination with minimal warning.

Which nursing action will be included in the plan of care for a patient with cirrhosis who has ascites and 4+ edema of the feet and legs? A) Restrict dietary protein intake B) Reposition the patient every 4 hours C) Use a pressure-relieving mattress D) Perform passive range of motion QID

C) Use a pressure-relieving mattress

With which clients does the nurse remain alert for the possibility of metabolic alkalosis? Select all that apply, letter only. A. client who has been NPO for 36 hours without fluid replacement B. Client receiving a rapid infusion of normal saline C. Client who has been self-managing indigestion with chronic ingestions of bicarbonate D. Client who has had continuous gastric suction for 48 hours E. Client having a sudden and severe asthma attack F. Client with uncontrolled diabetes mellitus

C, D

A patient with a sodium level of 115 is ordered to start Declomycin. Which statement is NOT true about this medication? A. "Declomycin is an antidiuretic hormone antagonist that treats SIADH." B. "Declomycin is part of the tetracycline family." C. "Declomycin is to be taken with food, preferably, milk." D. "Declomycin is contraindicated in children and pregnant/nursing women."

C. "Declomycin is to be taken with food, preferably, milk."

A patient with a sodium level of 115 is ordered to start Declomycin. Which statement is NOT true about this medication?* A. "Declomycin is an antidiuretic hormone antagonist that treats SIADH." B. "Declomycin is part of the tetracycline family." C. "Declomycin is to be taken with food, preferably, milk." D. "Declomycin is contraindicated in children and pregnant/nursing women."

C. "Declomycin is to be taken with food, preferably, milk."

A patient is being discharged after being hospitalized with a Magnesium level of 0.7. After diet teaching with this patient, which statement by the patient warrants you to re-educate the patient? A. "I love eating salads with kale and spinach". B. "I'm shocked mackerel contains magnesium". C. "I can't believe I have to give up dark chocolate". D. "Tonight I'm cooking fried potatoes with a side dish of peas".

C. "I can't believe I have to give up dark chocolate".

You have completed diet teaching with a patient who has hypernatremia. Which statement by the patient causes concern? A. "I will buy fresh vegetables and fruits." B. "I will avoid eating canned foods." C. "I'm glad I can still eat sandwiches with bologna." D. "I will avoid cooking with butter."

C. "I'm glad I can still eat sandwiches with bologna."

True or False: Gastric lesions and celiac disease cannot be diagnosed using an EGD

false

You have completed diet teaching with a patient who has hypernatremia. Which statement by the patient causes concern?* A. "I will buy fresh vegetables and fruits." B. "I will avoid eating canned foods." C. "I'm glad I can still eat sandwiches with bologna." D. "I will avoid cooking with butter."

C. "I'm glad I can still eat sandwiches with bologna."

A patient has a calcium level of 12.5. Which medication will most likely be ordered for this patient? A. Calcium Chloride B. 10% Calcium Gluconate C. Calcitonin D. Hydrochlorothiazide

C. Calcitonin

Which electrolyte is directly affected by the hormones secreted by the parathyroid? A. Potassium B. Sodium C. Calcium D. Magnesium

C. Calcium Parathyroid hormone (PTH) regulates serum calcium levels.

A patient is recovering from parathyroid surgery. Morning labs values are back. Which of the following lab values would correlate as a complication from this type of surgery? A. Calcium 8.7 B. Calcium 12.5 C. Calcium 6.9 D. Calcium 9.2

C. Calcium 6.9

A patient's calcium level is 11.2. Which option below could be the cause? A. None, 11.2 is a normal calcium level B. Cushing's Syndrome C. Hydrochlorothiazide D. Hypoparathyroidism

C. Hydrochlorothiazide

A patient has a sodium level of 119. Which of the following is NOT related to this finding? A. Over secretion of ADH (antidiuretic hormone) B. Low salt diet C. Inadequate water intake D. Hypotonic fluid infusion (overload)

C. Inadequate water intake

A patient has a sodium level of 119. Which of the following is NOT related to this finding?* A. Over secretion of ADH (antidiuretic hormone) B. Low salt diet C. Inadequate water intake D. Hypotonic fluid infusion (overload)

C. Inadequate water intake

A patient's magnesium level is 3.0. The cater associate brings the patient a dinner tray. Which item on the tray would you remove to ensure the patient does not eat? A. Macaroni B. Tomatoes C. Kale D. Onions

C. Kale

B (Cardiac effects may include sinus tachycardia, increased pulse pressure, and palpitations.)

Cardiac effects of hyperthyroidism include which of the following? a) Decreased systolic BP b) Palpitations c) Bradycardia d) Decreased pulse pressure

What is Primary Hypothyroidism?

Caused by destruction of thyroid tissue or defective hormone synthesis

A patient is admitted to the ER. The patient receives dialysis on Tuesdays and Thursdays of every week, and presents with a palpable AV shunt (thrill present) in the left upper arm. The patient is extremely lethargic and family members are present to help answer questions. While collecting the patient's medication history the daughter states her mother has been taking "a lot" of Maalox lately due to upset stomach. You note this to be a significant finding. Which of the following lab values correlates with this finding? A. Magnesium level of 1.0 B. Magnesium level of 2.4 C. Magnesium level of 3.6 D. Magnesium level of 1.4

C. Magnesium level of 3.6

Which patient is at risk for hyperkalemia? A. Patient with Parathyroid cancer B. Patient with Cushing's Syndrome C. Patient with Addison's Disease D. Patient with breast cancer

C. Patient with Addison's Disease

A patient has diabetes insipidus (DI). The LPN/LVN should anticipate giving which drug? A. Cortisone B. Florinef C. Pitressin D. Insulin

C. Pitressin Diabetes insipidus is managed by replacing fluid and electrolytes, along with hormone therapy, represents the basis of treatment of DI. In central DI, the hormone of choice to replace insufficient antidiuretic hormone (ADH) is desmopressin acetate (DDAVP), available orally, intravenously, or nasally. Other hormone medication choices exist, such as vasopressin (Pitressin) via nasal inhalation or by injection.

A patient is presenting with an orthostatic blood pressure of 80/40 when she stands up, thready and weak pulse of 58, and shallow respirations. In addition, the patient has been having frequent episodes of vomiting and nausea and is taking hydrochlorothiazide. Which of the following findings would explain the patient's condition? A. Potassium level of 7.0 B. Potassium level of 3.5 C. Potassium level of 2.4 D. None of the options are correct

C. Potassium level of 2.4

What is Secondary Hypothyroidism?

Caused by pituitary dysfunction with decreased TSH secretions or hypothalamic dysfunction with decreased TRH

The nurse is caring for a patient with Graves' disease. Which finding would indicate a complication of this patient's disease process? A. Extreme fatigue B. Tachycardia C. Shortness of breath D. Urinary frequency

C. Shortness of breath Primary hyperthyroidism is also known as Graves' disease or toxic goiter. A patient with Grave's disease (hyperthyroidism) who experiences shortness of breath is most likely experiencing respiratory and cardiac complications of the disease. A patient with hypothyroidism, not hyperthyroidism, experiences extreme fatigue. Although a patient with Grave's disease does experience tachycardia, it is a symptom of the disease process, not a complication of it. Urinary frequency is not associated with Grave's disease.

A patient with Cushing's Syndrome has been experiencing an infection and has a fever of 102'F. On assessment, you find the patient to be confused, restless, has dry mucous membranes, and flushed skin. Which finding below correlates with the presentation of this patient? A. Sodium level of 144 B. Sodium level of 115 C. Sodium level of 170 D. Sodium level of 135

C. Sodium level of 170

A patient with Cushing's Syndrome has been experiencing an infection and has a fever of 102'F. On assessment, you find the patient to be confused, restless, has dry mucous membranes, and flushed skin. Which finding below correlates with the presentation of this patient?* A. Sodium level of 144 B. Sodium level of 115 C. Sodium level of 170 D. Sodium level of 135

C. Sodium level of 170

Which arrhythmia is a patient who has a Mg+ level of 0.8 most likely to experience? A. Heart block B. Bradycardia C. Torsades de pointes D. Normal sinus rhythm

C. Torsades de pointes

A patient has been admitted for management of hypoparathyroidism. The nurse should anticipate an order for: A. potassium. B. magnesium. C. calcium. D. iron.

C. calcium. The parathyroid is responsible for calcium and phosphorus absorption. Patients with hypoparathyroidism develop hypocalcemia.

Which of the following is clinical manifestation of cholelithiasis? a) Clay-colored stools b) Epigastric distress prior to a meal c) Nonpalpable abdominal mass d) Upper left quadrant abdominal pain

Clay-colored stools The patient with gallstones has clay-colored stools, and excruciating upper right quadrant pain that radiates to the back or right shoulder. The excretion of bile pigments by the kidneys makes urine very dark. The feces, no longer colored with bile pigments, are grayish (like putty) or clay colored. The patient develops a fever and may have a palpable abdominal mass.

With which client does the nurse remain alert for and assess most frequently for signs and symptoms of hypokalemia to prevent harm? A. 72-year-old taking the diuretic spironolactone for control of hypertension B. 62-year-old receiving IV solution of Ringer's lactate at a rate of 200 mL/hr C. 42-year-old trauma victim receiving a third infusion of packed red blood cells in 12 hours. D. 22-year-old receiving an IV infusion of regular insulin to manage an episode of ketoacidosis.

D

True or False: The diagnosis of acute gastritis is based solely on clinical symptoms

false - complete regeneration/healing occurs within a few days

A nurse is assessing a client who is dehydrated. Which of the following findings should the nurse expect? A. moist skin B. Distended neck veins C. Increased urinary output D. Tachycardia

D

A nurse is caring for a client in a long-term care facility who has become weak, confused, and experienced dizziness when standing. The clients temperature is 39.3C (100.9F), pulse 92/min, respirations 20/min, and blood pressure 108/60 mm Hg. Which of the following actions should the nurse take? A. Initiate fluid restrictions to limit intake B. Check for peripheral edema C. Encourage the client to ambulate to promote oxygenation D. Monitor for orthostatic hypotension

D

A nurse is obtaining arterial blood gasses for a client who has vomited for 24 hours. The nurse should expect which of the following acid-base imbalances to result from vomiting for 24 hours? A. Respiratory acidosis B. Respiratory alkalosis C. Metabolic acidosis D. Metabolic alkalosis

D

A patient with mild hypokalemia caused by a diuretic used is discharged home. Which intervention would the RN delegate to the home health aide. A. Instruction on proper use of the drugs B. Education about potassium-rich foods C. Assessment of muscle tone and strength D. Measurement of the patient's urine output

D

For which classification of drug would the nurse instruct a patient to increased intake of dietary potassium? A. Beta blockers B. Corticosteroids C. Alpha antagonists D. Loop diuretics

D

Which assessment finding would require priority nursing interventions in a patient with metabolic or respiratory acidosis? A. Dry skin B. Rapid respiratory rate C. Lethargy and confusion D. Bradycardia with widened QRS complex

D

Which electrolyte abnormality would the nurse anticipate when reviewing laboratory data for a patient admitted with metabolic acidosis? A. Hyponatremia B. Hypernatremia C. Hypokalemia D. Hyperkalemia

D

Which intervention would the nurse expect to be prescribed for a patient with hyponatremia? A. 2-g sodium diet B. Administration of furosemide C. IV administration of 0.45% normal saline D. Small-volume IV infusions of 3% normal saline

D

Which laboratory value requires the nurse to notify the primary health care provider? A. Sodium 133 mEq/L B. Calcium 9.0 mg/dL C. Potassium 3.6 mEq/L D. Magnesium 4.2 mEq/L

D

Which newly written prescription does the nurse administer first? A. oral calcium supplements to a patient with severe osteoporosis B. Oral phosphorus supplements to a patient with acute hypophosphatemia C. IV normal saline to a patient with serum sodium level of 134 mEq/L D. Oral potassium chloride to a patient whose serum potassium level is 3 mEq/L

D

Which prescribed action for a patient with serum sodium level of 149 mEq/L would the nurse clarify with the health care provider? A. Weigh the patient daily B. Monitor I & O C. Institute seizure precautions D. Ensure NPO status

D

The nurse is caring for a patient with hyperparathyroidism. What level of activity would the nurse expect to promote? A) Complete bed rest B) Bed rest with bathroom privileges C) Out of bed (OOB) to the chair twice a day D) Ambulation and activity as tolerated

D) Ambulation and activity as tolerated

10. The goal of nursing care of the client with an indwelling catheter and continuous drainage is largely directed at preventing infection of the urinary tract and encouraging urinary flow through the drainage system. Which of the following interventions encouraged by nurses working with these clients would not be appropriate in meeting this goal? A) Having the client drink up to 2000 mL per day B) Encouraging the client to eat foods that increase the acid in the urine C) Routine hygienic care D) Changing indwelling catheters every 72 hours.

D) Changing indwelling catheters every 72 hours.

9. Urinary incontinence is not a normal part of aging. An intervention used by nurses to assist clients to regain or maintain continence with individuals suffering from this problem would not include: A) Bladder training B) Habit training C) Prompted voiding D) Fluid restriction

D) Fluid restriction

What manifestation in the patient does the nurse recognize as an early sign of hepatic encephalopathy? A) Asterixis B) Becomes unconscious C) Has increasing oliguria D) Is irritable and lethargic

D) Is irritable and lethargic

6. The nurse recognizes that urinary elimination changes may occur even in healthy older adults because of which of the following? A) The bladder distends and its capacity increases. B) Older adults ignore the need to void. C) Urine becomes more concentrated. D) The amount of urine retained after voiding increases.

D) The amount of urine retained after voiding increases.

2. The primary goal of nursing care for a client with stress incontinence is to: A) Help the client adjust to the frequent episodes of incontinence. B) Eliminate all episodes of incontinence. C) Prevent the development of urinary tract infections. D) Decrease the number of incontinence episodes.

D)Decrease the number of incontinence episodes.

A patient is being discharged home after hospitalization with hypocalcemia. Which statement by the patient indicates she understood the dietary instructions? A. "I will avoid sardines. B. "I'll avoid salt and Vitamin-D supplements." C. "I will tell my husband to only purchase skim milk." D. "I will be sure to eat lots of cheese, tofu and spinach."

D. "I will be sure to eat lots of cheese, tofu and spinach."

Which of the following patients is MOST at risk for hypermagnesemia? A. A patient with alcoholism B. A patient taking a proton-pump inhibitor called Protonix C. A patient suffering from Crohn's Disease D. A patient with a magnesium level of 0.6 receiving IV magnesium sulfate

D. A patient with a magnesium level of 0.6 receiving IV magnesium sulfate

What should be kept at the bedside of a patient immediately after thyroidectomy? A. A ventilator B. An endotracheal tube C. An airway D. A tracheostomy tray

D. A tracheostomy tray Laryngeal swelling is not uncommon in a patient after a thyroidectomy. A tracheostomy tray should be immediately available. A ventilator is not necessary, and the endotracheal tube will be very difficult, if not impossible, to intubate if edema has already occurred. An airway will be of no value, since the edema will be in the trachea.

A patient has a Magnesium level of 1.3. Which of the following is NOT a sign or symptom of this condition? A. Tall T-wave and depressed ST segment B. Torsades de pointes C. Positive Trouesseau's and Chvostek's D. Absent deep tendon reflexes

D. Absent deep tendon reflexes

Which of the following is not a contributor to a condition of ascites? A. Elevated levels of aldosterone B. Hypertension C. Low levels of albumin D. Elevated levels of angiotension I

D. Elevated levels of angiotension I

A patient with a sodium level of 178 is ordered to be started on 0.45% Saline. What is the most IMPORTANT nursing intervention for this patient? A. Maintain patent IV B. Give rapidly to ensure fluids levels are shifted properly C. Clarify doctor's order because 0.45% saline is contraindicated in hypernatremia D. Give slowly and watch for signs and symptoms of cerebral edema

D. Give slowly and watch for signs and symptoms of cerebral edema

A patient with a sodium level of 178 is ordered to be started on 0.45% Saline. What is the most IMPORTANT nursing intervention for this patient?* A. Maintain patent IV B. Give rapidly to ensure fluids levels are shifted properly C. Clarify doctor's order because 0.45% saline is contraindicated in hypernatremia D. Give slowly and watch for signs and symptoms of cerebral edema

D. Give slowly and watch for signs and symptoms of cerebral edema

A patient's lab work shows that they have a high parathyroid hormone level. Which condition is the patient at risk for? A. Hyperkalemia B. Hypocalcemia C. Hypokalemia D. Hypercalcemia

D. Hypercalcemia

Tall peaked T-waves, flat P-waves, prolonged PR intervals and widened QRS complexes can present in which of the following conditions? A. Hypocalemia B. Hypercalemia C. Hypokalemia D. Hyperkalemia

D. Hyperkalemia

A patient with a magnesium level of 3.6 would exhibit which of the signs and symptoms EXCEPT? A. Hypotension B. Profound Lethargy C. Respiratory failure D. Hyperreflexia of the deep tendons

D. Hyperreflexia of the deep tendons

Which patient below is at risk for experiencing Hypovolemic Hyponatremia? A. Patient with congestive heart failure B. Patient with cirrhosis of the liver C. Patient on IV saline at 250 cc/hr D. Patient with nasogastric tube suction experiencing diarrhea

D. Patient with nasogastric tube suction experiencing diarrhea

Which patient below is at risk for experiencing Hypovolemic Hyponatremia?* A. Patient with congestive heart failure B. Patient with cirrhosis of the liver C. Patient on IV saline at 250 cc/hr D. Patient with nasogastric tube suction experiencing diarrhea

D. Patient with nasogastric tube suction experiencing diarrhea

A patient has a calcium level of 7.2. What sign below is indicative of this lab value? A. None this is a normal calcium level B. Shortened ST segment C. Hypoactive bowel sounds D. Prolonged QT interval on the EKG

D. Prolonged QT interval on the EKG

Which of the following is indicative of an EKG change in a case of hypokalemia? A. Widened QRS complex and prolonged PR interval B. Prolonged ST interval and Widened T-wave C. Tall T-waves and depressed ST segment D. ST depression and inverted T-wave

D. ST depression and inverted T-wave

Which of the following is not a cause of hypocalcemia? A. Low parathyroid hormone B. Crohn's Disease C. Acute Pancreatitis D. Thiazide Diuretics

D. Thiazide Diuretics

True or False: It is okay to have bedtime snacks with PUD as long as you sleep with HOB elevate 6 inches

false --> bedtime snacks stimulate gastric acid secretion

A 70-year-old patient is admitted with acute pancreatitis. The nurse understands that the mortality rate associated with acute pancreatitis increases with advanced age and attributes this to which of the following gerontologic considerations associated with the pancreas? a) Decreases in the physiologic function of major organs b) Increases in the rate of pancreatic secretion c) Development of local complications d) Increases in the bicarbonate output by the kidneys

Decreases in the physiologic function of major organs Acute pancreatitis affects people of all ages, but the mortality rate associated with acute pancreatitis increases with advancing age. The pattern of complications changes with age. Younger patients tend to develop local complications; the incidence of multiple organ failure increases with age, possibly as a result of progressive decreases in physiologic function of major organs with increasing age.

What is hypothyroidism?

Deficiency of thyroid hormone that causes general slowing metabolic rate

What should you not take levothyroxine with and why?

Do not take within 4 hours of any GI medications because it can decrease the absorption of thyroid medication

B (With the diagnosis of hypothyroidism, extreme fatigue makes it difficult for the person to complete a full day's work or participate in usual activities.)

During an assessment of a patient's functional health pattern, which question by the nurse directly addresses the patient's thyroid function? a) "Do you have to get up at night to empty your bladder?" b) "Do you experience fatigue even if you have slept a long time?" c) "Have you experienced any headaches or sinus problems?" d) "Can you describe the amount of stress in your life?"

Increased appetite and thirst may indicate that a client with chronic pancreatitis has developed diabetes melitus. Which of the following explains the cause of this secondary diabetes? a) Dysfunction of the pancreatic islet cells b) Ingestion of foods high in sugar c) Inability for the liver to reabsorb serum glucose d) Renal failure

Dysfunction of the pancreatic islet cells Diabetes mellitus resulting from dysfunction of the pancreatic islet cells is treated with diet, insulin, or oral antidiabetic agents. The hazard of severe hypoglycemia with alcohol consumption is stressed to the client and family. When secondary diabetes develops in a client with chronic pancreatitis, the client experiences increased appetite, thirst, and urination. A standard treatment with pancreatitis is to make the client NPO. The dysfunction is related to the pancreas, not the liver.

A nurse is assessing a client who has pancreatitis. The client's arterial blood gases reveal metabolic acidosis. Which of the following are expected findings? Select all that apply, letter only. A. Tachycardia B. Hypertension C. Bounding pulses D. Hyperreflexia E. Dysrhythmia F. Tachypnea

E, F

What is a visual examination of the esophagus, stomach, and duodenum?

EGD

What is the gold standard for diagnosing gastritis?

EGD

what is the major diagnostic test used for PUD?

EGD

B (Thyroid crisis usually occurs in the first 12 hours after thyroidectomy and causes exaggerated signs of hyperthyroidism, such as high fever, tachycardia, and extreme restlessness.)

Early this morning, a client had a subtotal thyroidectomy. During evening rounds, the nurse assesses the client, who now has nausea, a temperature of 105° F (40.5° C), tachycardia, and extreme restlessness. What is the most likely cause of these signs? a) Tetany b) Thyroid crisis c) Diabetic ketoacidosis d) Hypoglycemia

A patient is diagnosed with mild acute pancreatitis. What does the nurse understand is characteristic of this disorder? a) Edema and inflammation b) Pleural effusion c) Sepsis d) Disseminated intravascular coagulopathy

Edema and inflammation Mild acute pancreatitis is characterized by edema and inflammation confined to the pancreas. Minimal organ dysfunction is present, and return to normal function usually occurs within 6 months.

What is overt hypothyroidism?

Elevated TSH and decreased T4

A client is admitted to the healthcare facility suspected of having acute pancreatitis and undergoes laboratory testing. Which of the following would the nurse expect to find? a) Increased serum calcium levels b) Decreased white blood cell count c) Elevated urine amylase levels d) Decreased liver enzyme levels

Elevated urine amylase levels Elevated serum and urine amylase, lipase, and liver enzyme levels accompany significant pancreatitis. If the common bile duct is obstructed, the bilirubin level is above normal. Blood glucose levels and white blood cell counts can be elevated. Serum electrolyte levels (calcium, potassium, and magnesium) are low.

genes that were once protective against long periods when food was not available are now maladaptive in societies in which food availability is no longer an issue

Energy thrifty genes

True or false: PPIs should be discontinued immediately if signs of hemorrhage appear

FALSE - PPIs should NOT be stopped abruptly to prevent rebound activation of the proton pump

A (During stressful procedures or significant illnesses, additional supplementary therapy with glucocorticoids is required to prevent addisonian crisis. Physical activity, dehydration and vaccine administration would not normally be sufficiently demanding such to require glucocorticoids.)

Following an addisonian crisis, a patients adrenal function has been gradually regained. The nurse should ensure that the patient knows about the need for supplementary glucocorticoid therapy in which of the following circumstances? A) Episodes of high psychosocial stress B) Periods of dehydration C) Episodes of physical exertion D) Administration of a vaccine

A (Thyroid hormone antagonists, which block thyroid hormone synthesis, combat increased production of thyroid hormone. Treatment of hyperthyroidism also may include radioiodine therapy, which destroys some thyroid gland cells, and surgery to remove part of the thyroid gland; both treatments decrease thyroid hormone production.)

For a client with hyperthyroidism, treatment is most likely to include: a) a thyroid hormone antagonist. b) a synthetic thyroid hormone. c) thyroid extract. d) emollient lotions.

Which of the following conditions in a patient with pancreatitis makes it necessary for the nurse to check fluid intake and output, hourly urine output, and monitor electrolyte levels? a) Frequent vomiting leading to loss of fluid volume b) High glucose levels in the blood c) Dry mouth, which makes the patient thirsty d) Acetone in the urine

Frequent vomiting leading to loss of fluid volume Fluid and electrolyte disturbances are common complications because of nausea, vomiting, movement of fluid from the vascular compartment to the peritoneal cavity, diaphoresis, fever, and the use of gastric suction. The nurse assesses the patient's fluid and electrolyte status by noting skin turgor and moistness of mucous membranes. The nurse weighs the patient daily and carefully measures fluid intake and output, including urine output, nasogastric secretions, and diarrhea.

A nurse is preparing a client for surgery. During preoperative teaching, the client asks where is bile stored. The nurse knows that bile is stored in the: a) Gallbladder b) Duodenum c) Cystic duct d) Common bile duct

Gallbladder The gallbladder functions as a storage depot for bile.

What are most gastric and duodenal ulcers caused by?

H.pylori infection

What does rapid urease testing detect?

H.pylori infection

Which drug class *decreases gastric acid secretion by blocking histamine receptors in parietal cells*?

H2 antagonists (blockers)

What type of drugs are usually prescribed to block gastric secretions in patients with gastritis?

H2 receptor atagonists (famotidine)

Patients with Graves disease

Have enlarged thyroid gland called a goiter can be a result of excess TSH.

Upon receiving the dinner tray for a client admitted with acute gallbladder inflammation, the nurse will question which of the following foods on the tray? a) Vanilla pudding b) Hot roast beef sandwich with gravy c) White rice d) Mashed potatoes

Hot roast beef sandwich with gravy The diet immediately after an episode of acute cholecystitis is initially limited to low-fat liquids. Cooked fruits, rice or tapioca, lean meats, mashed potatoes, bread, and coffee or tea may be added as tolerated. The client should avoid fried foods such as fried chicken, because fatty foods may bring on an episode of cholecystitis.

What histamine blocker(s) can be given *IV* for the prevention of *surgical stress ulcers*?

IV famotidine and IV ranitidine

What is the treatment for myxedema coma?

IV thyroid hormone

What does myxedema come result in?

Impaired consciousness -Subnormal temperature, hypotension, hypoventilation - Cardiovascular collapse

A student nurse is preparing a plan of care for a client with chronic pancreatitis. What nursing diagnosis related to the care of a client with chronic pancreatitis is the priority? a) Impaired nutrition: less than body requirements b) Nausea c) Anxiety d) Disturbed body image

Impaired nutrition: less than body requirements While each diagnosis may be applicable to this client, the priority nursing diagnosis is impaired nutrition: less than body requirements. The physician, nurse, and dietitian emphasize to the client and family the importance of avoiding alcohol and foods that have produced abdominal pain and discomfort in the past. Oral food or fluid intake is not permitted during the acute phase.

Why should *flavored antacids* be AVOIDED?

flavoring increases the emptying time of the stomach so desired effect is negated

Thyroid storm

Is a hypothyroidism crisis where the levels of thyroid hormone become toxic. It is a life-threatening emergency

Which of the following enzymes aids in the digestion of fats? a) Secretin b) Trypsin c) Lipase d) Amylase

Lipase Lipase is a pancreatic enzyme that aids in the digestion of fats. Amylase aids in the digestion of carbohydrates. Secretin is responsible for stimulating secretion of pancreatic juice. Trypsin aids in the digestion of protein.

Respiratory Clinical Manifestations of Hypothyroidism

Low exercise tolerance - Shortness of breath on exertion - Because the body is not keeping up with its demands

C (Pituitary disorders usually result from excessive or deficient production and secretion of a specific hormone. Dwarfism occurs when secretion of growth hormone is insufficient during childhood. Dwarfism occurs when secretion of growth hormone from the pituitary gland is insufficient during childhood.)

Lydia Kranston, a 3-year-old female, is being seen by a healthcare provider in the endocrinology group where you practice nursing. She has a significant height deficit and is to be evaluated for diagnostic purposes. What could be the cause of her disorder? a) Thyroid disorder b) Adrenal disorder c) Pituitary disorder d) Parathyroid disorder

A critical care nurse is caring for a client with acute pancreatitis. One potentially severe complication involves the respiratory system. Which of the following would be an appropriate intervention to prevent complications associated with the respiratory system? a) Maintain the client in a semi-Fowler's position. b) Administer enteral or parenteral nutrition. c) Withhold oral feedings. d) Carry out wound care as prescribed.

Maintain the client in a semi-Fowler's position. The nurse maintains the client in the semi-Fowler's position to decrease pressure on the diaphragm by a distended abdomen and to increase respiratory expansion. Respiratory distress and hypoxia are common, and the client may develop diffuse pulmonary infiltrates, dyspnea, tachypnea, and abnormal blood gas values. The client who has undergone surgery may have multiple drains or an open surgical incision and is at risk for skin breakdown and infection. Oral food or fluid intake is not permitted; therefore, enteral or parenteral feedings may be prescribed.

Produced in the hypothalamus to suppress appetite

Neuropeptide Y

Which of the following would be included as a postoperative intervention for the patient undergoing a laparoscopic cholecystectomy? a) Semi-Fowler's position b) Observe color of sclera c) Low-carbohydrate, low-protein diet immediately after surgery d) NPO status postop for 2 days

Observe color of sclera The nurse should be particularly observant of the color of the sclera. After recovery from anesthesia, the patient is placed in the low Fowler's position. Water and other fluids may be administered within hours after laparoscopic procedures. A soft diet is started after bowel sounds return.

What is subclinical hypothyroidism?

Occurs when the TSH is greater then 4.5 mU/L and has normal T4 levels

D (Hypocalcemia may follow thyroid surgery if the parathyroid glands were removed accidentally. Signs and symptoms of hypocalcemia may be delayed for up to 7 days after surgery.)

On the third day after a partial thyroidectomy, a client exhibits muscle twitching and hyperirritability of the nervous system. When questioned, the client reports numbness and tingling of the mouth and fingertips. Suspecting a life-threatening electrolyte disturbance, the nurse notifies the surgeon immediately. Which electrolyte disturbance most commonly follows thyroid surgery? a) Hypermagnesemia b) Hyperkalemia c) Hyponatremia d) Hypocalcemia

What is the common drug regimen for the treatment of H.pylori?

PPI-triple therapy for 10-14 days: -PPI (lansoprazole) -two ab'tics (metronidazole + tetracycline) or (clarithromycin + amoxicillin)

What is the drug class of choice for treating patients with acid-related disorders?

PPIs

What type of drugs are prescribed for patients with gastritis to suppress gastric acid secretion?

PPIs

Which drug class *suppresses gastric acid secretion and is indicated for short-term and long-term use for PUD, symptomatic heartburn, and H.pylori treatment*?

PPIs *give 30 min before the main meal of the day*

What results when mucosal defenses become impaired and no longer protect the epithelium from the effects of acid and pepsin?

PUD

A patient is admitted to the health care center with hyperglycemia, a 15-pound weight loss, and complaints of vague upper and midabdominal pain that increases in intensity at night. His health history record indicates that he is an alcoholic, smokes a pack of cigarettes daily, and has had diabetes for the past 20 years. On examination, the nurse finds swelling in his feet and abdominal ascites. Based on the clinical manifestations, which of the following is most likely to be the diagnosis? a) Cholecystitis b) Pancreatic pseudocysts c) Pancreatic carcinoma d) Acute pancreatitis with edema

Pancreatic carcinoma Pain, jaundice, and weight loss are considered classic signs of pancreatic carcinoma. Other signs include rapid, profound, and progressive weight loss as well as vague upper or midabdominal pain or discomfort unrelated to any gastrointestinal function and it is often difficult to describe. It is often more severe at night and is accentuated when lying supine. The formation of ascites is common. An important sign is the onset of symptoms of insulin deficiency: glucosuria, hyperglycemia, and abnormal glucose tolerance. Therefore, diabetes may be an early sign of carcinoma of the pancreas.

Exophthalmos

Patients with Graves disease have accumulation of inflammation behind orbital tissue the make the eyes look buggy

Proptosus

Patients with Graves disease have forward displacement of eyes.

Nursing care of patients with hyperthyroidism

Provide light bedding if patient is diaphoretic, restrict visitors, prepare for thyroid surgery if required, post-operative care after surgery, maintain vitals

Treatment of hyper thyroidism

Radioactive iodine therapy, thyroidectomy, anti thyroid medications

What is hyperthyroidism

Sustained increase in synthesis and release of thyroid hormones. More rare. Highest in patients 30 to 50 years old. Common forms Graves disease.

A client is admitted to the health care facility with abdominal pain, a low-grade fever, abdominal distention, and weight loss. The physician diagnoses acute pancreatitis. What is the primary goal of nursing care for this client? a) Maintaining adequate nutritional status b) Preventing fluid volume overload c) Teaching about the disease and its treatment d) Relieving abdominal pain

Relieving abdominal pain The predominant clinical feature of acute pancreatitis is abdominal pain, which usually reaches peak intensity several hours after onset of the illness. Therefore, relieving abdominal pain is the nurse's primary goal. Because acute pancreatitis causes nausea and vomiting, the nurse should try to prevent fluid volume deficit, not overload. The nurse can't help the client achieve adequate nutrition or understand the disease and its treatment until the client is comfortable and no longer in pain.

A client being treated for pancreatitis faces the risk of atelectasis. Which of the following interventions would be important to implement to minimize this risk? a) Instruct the client to avoid coughing. b) Withhold oral feedings for the client. c) Monitor pulse oximetry every hour. d) Reposition the client every 2 hours.

Reposition the client every 2 hours. Repositioning the client every 2 hours minimizes the risk of atelectasis in a client who is being treated for pancreatitis. The client should be instructed to cough every 2 hours to reduce atelectasis. Monitoring the pulse oximetry helps show changes in respiratory status and promote early intervention, but it would do little to minimize the risk of atelectasis. Withholding oral feedings limits the reflux of bile and duodenal contents into the pancreatic duct.

Symptoms of thyroid storm

Restlessness, agitation, seizures, tachycardia, high fever, hypertension Treated with the anti thyroid drug PTU radioactive iodine or beta blockers

The nursing student has just reviewed material in the course textbook regarding pancreatitis. The student knows that a major symptom of pancreatitis that causes the client to seek medical care is: a) Jaundice b) Mental agitation c) Severe abdominal pain d) Fever

Severe abdominal pain The pain of acute pancreatitis is often very severe, necessitating the liberal use of analgesics. Jaundice is indicative of a bile duct obstruction. Fever and mental agitation are not indicative of pancreatitis.

How is Hepatitis E transmitted? A. Fecal-oral B. Percutaneous C. Mucosal D. Body fluids

The answer is A.

A patient has lab work drawn and it shows a positive HBsAg. What education will you provide to the patient? A. Avoid sexual intercourse or intimacy such as kissing until blood work is negative. B. The patient is now recovered from a previous Hepatitis B infection and is now immune. C. The patient is not a candidate from antiviral or interferon medications. D. The patient is less likely to develop a chronic infection.

The answer is A. A positive HBsAg (hepatitis B surface antigen) indicates an active Hepatitis B infection. Therefore, the patient should avoid sexual intercourse and other forms of intimacy until their HBsAg is negative.

A patient with viral Hepatitis states their flu-like symptoms have subsided. However, they now have yellowing of the skin and sclera along with dark urine. Based on this finding, this is what phase of Hepatitis? A. Icteric B. Posticteric C. Preicteric D. Convalescent

The answer is A. The Preicteric (prodromal) Phase: flulike symptoms...joint pain, fatigue, nausea vomiting, abdominal pain change in taste, liver enzymes and bilirubin increasing....Icteric Phase: decrease in the flu-like symptoms but will have jaundice and dark urine (buildup of bilirubin) yellowing of skin and white part of the eyeball, clay-colored stool (bilirubin not going to stool to give it's normal brown color) enlarged liver and pain in this area....Posticteric (convalescent) Phase: jaundice and dark urine start to subside and stool returns to normal brown color, liver enzymes and bilirubin decrease to normal

A patient with Hepatitis is extremely confused. The patient is diagnosed with Hepatic Encephalopathy. What lab result would correlate with this mental status change? A. Ammonia 100 mcg/dL B. Bilirubin 7 mg/dL C. ALT 56 U/L D. AST 10 U/L

The answer is A. When ammonia levels become high (normal 15-45 mcg/dL) it affects brain function. Therefore, the nurse would see mental status changes in a patient with this ammonia level.

A patient was exposed to Hepatitis B recently. Postexposure precautions include vaccination and administration of HBIg (Hepatitis B Immune globulin). HBIg needs to be given as soon as possible, preferably ___________ after exposure to be effective. A. 2 weeks B. 24 hours C. 1 month D. 7 days

The answer is B. HBIg should be given 24 hours after exposure to maximum effectiveness of temporary immunity against Hepatitis B. It would be given within 12 hours after birth to an infant born to a mother who has Hepatitis B.

Which patient below is at MOST risk for developing a complication related to a Hepatitis E infection? A. A 45-year-old male with diabetes. B. A 26-year-old female in the 3rd trimester of pregnancy. C. A 12-year-old female with a ventricle septal defect. D. A 63-year-old male with cardiovascular disease.

The answer is B. Patients who are in the 3rd trimester of pregnancy are at a HIGH risk of developing a complication related to a Hepatitis E infection.

The liver receives blood from two sources. The _____________ is responsible for pumping blood rich in nutrients to the liver. A. hepatic artery B. hepatic portal vein C. mesenteric artery D. hepatic iliac vein

The answer is B. The liver receives blood from two sources. The hepatic portal vein is responsible for pumping blood rich in nutrients to the liver.

A 36-year-old patient's lab work show anti-HAV and IgG present in the blood. As the nurse you would interpret this blood work as? A. The patient has an active infection of Hepatitis A. B. The patient has recovered from a previous Hepatitis A infection and is now immune to it. C. The patient is in the preicetric phase of viral Hepatitis. D. The patient is in the icteric phase of viral Hepatitis.

The answer is B. When a patient has anti-HAV (antibodies of the Hepatitis A virus) and IgG, this means the patient HAD a past infection of Hepatitis A but it is now gone, and the patient is immune to Hepatitis A now. If the patient had anti-HAV and IgM, this means the patient has an active infection of Hepatitis A.

What is the BEST preventive measure to take to help prevent ALL types of viral Hepatitis? A. Vaccination B. Proper disposal of needles C. Hand hygiene D. Blood and organ donation screening

The answer is C. Hand hygiene can help prevent all types of viral hepatitis. However, not all types of viral Hepatitis have a vaccine available or are spread through needle sticks or blood/organs donations. Remember Hepatitis A and E are spread only via fecal-oral routes.

The physician writes an order for the administration of Lactulose. What lab result indicates this medication was successful? A. Bilirubin <1 mg/dL B. ALT 8 U/L C. Ammonia 16 mcg/dL D. AST 10 U/L

The answer is C. Lactulose is ordered to decrease a high ammonia level. It will cause excretion of ammonia via the stool. A normal ammonia level would indicate the medication was successful (normal ammonia level 15-45 mcg/dL).

A patient with Hepatitis A asks you about the treatment options for this condition. Your response is? A. Antiviral medications B. Interferon C. Supportive care D. Hepatitis A vaccine

The answer is C. There is no current treatment for Hepatitis A but supportive care and rest. Treatments for the other types of Hepatitis such as B, C, and D include antiviral or interferon (mainly the chronic cases) along with rest.

A patient is prescribed Peginterferon alfa-2a. The nurse will prepare to administer this medication what route? A. Oral B. Intramuscular C. Subcutaneous D. Intravenous

The answer is C. This medication is administered subq.

A patient has completed the Hepatitis B vaccine series. What blood result below would demonstrate the vaccine series was successful at providing immunity to Hepatitis B? A. Positive IgG B. Positive HBsAg C. Positive IgM D. Positive anti-HBs

The answer is D. A positive anti-HBs (Hepatitis B surface antibody) indicates either a past infection of Hepatitis B that is now cleared and the patient is immune, OR that the vaccine has been successful at providing immunity. A positive HBsAg (Hepatitis B surface antigen) indicates an active infection.

A patient is diagnosed with Hepatitis A. The patient asks how a person can become infected with this condition. You know the most common route of transmission is? A. Blood B. Percutaneous C. Mucosal D. Fecal-oral

The answer is D. Hepatitis A is most commonly transmitted via the fecal-oral route.

What is the MOST common transmission route of Hepatitis C? A. Blood transfusion B. Sharps injury C. Long-term dialysis D. IV drug use

The answer is D. IV drug use is the MOST common transmission route of Hepatitis C.

A 25-year-old patient was exposed to the Hepatitis A virus at a local restaurant one week ago. What education is important to provide to this patient? A. Inform the patient to notify the physician when signs and symptoms of viral Hepatitis start to appear. B. Reassure the patient the chance of acquiring the virus is very low. C. Inform the patient it is very important to obtain the Hepatitis A vaccine immediately to prevent infection. D. Inform the patient to promptly go to the local health department to receive immune globulin.

The answer is D. Since the patient was exposed to Hepatitis A, the patient would need to take preventive measures to prevent infection because infection is possible. The patient should not wait until signs and symptoms appear because the patient can be contagious 2 weeks BEFORE signs and symptoms appear. The vaccine would not prevent Hepatitis A from this exposure, but from possible future exposures because it takes the vaccine 30 days to start working. The best answer is option D. The patient would need to receive immune globulin to provide temporary immunity within 2 weeks of exposure.

TRUE or FALSE: A patient with Hepatitis A is contagious about 2 weeks before signs and symptoms appear and 1-3 weeks after the symptoms appear.

The answer is TRUE.

Select all the types of viral Hepatitis that have preventive vaccines available in the United States? A. Hepatitis A B. Hepatitis B C. Hepatitis C D. Hepatitis D E. Hepatitis E

The answers are A and B. Currently there is only a vaccine for Hepatitis A and B in the U.S.

A patient is diagnosed with Hepatitis D. What statement is true about this type of viral Hepatitis? Select all that apply: A. The patient will also have the Hepatitis B virus. B. Hepatitis D is most common in Southern and Eastern Europe, Mediterranean, and Middle East. C. Prevention of Hepatitis D includes handwashing and the Hepatitis D vaccine. D. Hepatitis D is most commonly transmitted via the fecal-oral route.

The answers are A and B. These are true statements about Hepatitis D. Prevention for Hepatitis D includes handwashing and the Hepatitis B vaccine (since it occurs only with the Hepatitis B virus). It is transmitted via blood.

Which statements are INCORRECT regarding the anatomy and physiology of the liver? Select all that apply: A. The liver has 3 lobes and 8 segments. B. The liver produces bile which is released into the small intestine to help digest fats. C. The liver turns urea, a by-product of protein breakdown, into ammonia. D. The liver plays an important role in the coagulation process.

The answers are A and C. The liver has 2 lobes (not 3), and the liver turns ammonia (NOT urea), which is a by-product of protein breakdown, into ammonia. All the other statements are true about liver's anatomy and physiology.

You're providing an in-service on viral hepatitis to a group of healthcare workers. You are teaching them about the types of viral hepatitis that can turn into chronic infections. Which types are known to cause ACUTE infections ONLY? Select all that apply: A. Hepatitis A B. Hepatitis B C. Hepatitis C D. Hepatitis D E. Hepatitis E

The answers are A and E. Only Hepatitis A and E cause ACUTE infections...not chronic. Hepatitis B, C, and D can cause both acute and chronic infections.

Select all the signs and symptoms associated with Hepatitis? A. Arthralgia B. Bilirubin 1 mg/dL C. Ammonia 15 mcg/dL D. Dark urine E. Vision changes F. Yellowing of the sclera G. Fever H. Loss of appetite

The answers are A, D, F, G, and H. The bilirubin and ammonia levels are normal in these options, but they would be abnormal in Hepatitis. A normal bilirubin is 1 or less, and a normal ammonia is 15-45 mcg/dL.

During the posticteric phase of Hepatitis the nurse would expect to find? Select all that apply: A. Increased ALT and AST levels along with an increased bilirubin level B. Decreased liver enzymes and bilirubin level C. Flu-like symptoms D. Resolved jaundice and dark urine

The answers are B and D. Posticteric (convalescent) Phase: jaundice and dark urine start to subside and stool returns to normal brown color, liver enzymes and bilirubin decrease to normal

Select all the ways a person can become infected with Hepatitis B: A. Contaminated food/water B. During the birth process C. IV drug use D. Undercooked pork or wild game E. Hemodialysis F. Sexual intercourse

The answers are B, C, E, and F. Hepatitis B is spread via blood and body fluids. It could be transmitted via the birthing process, IV drug use, hemodialysis, or sexual intercourse etc.

Which patients below are at risk for developing complications related to a chronic hepatitis infection, such as cirrhosis, liver cancer, and liver failure? Select all that apply: A. A 55-year-old male with Hepatitis A. B. An infant who contracted Hepatitis B at birth. C. A 32-year-old female with Hepatitis C who reports using IV drugs. D. A 50-year-old male with alcoholism and Hepatitis D.

The answers are B, C, and D. Infants or young children who contract Hepatitis B are at a very high risk of developing chronic Hepatitis B (which is why option B is correct). Option C is correct because most cases of Hepatitis C turn into chronic cases and IV drug use increases this risk even more. Option D is correct because Hepatitis D occurs when Hepatitis B is present and constant usage of alcohol damages the liver. Therefore, the patient is at high risk of developing chronic hepatitis. Hepatitis A and E tend to only cause acute infections....not chronic.

A patient with Hepatitis has a bilirubin of 6 mg/dL. What findings would correlate with this lab result? Select all that apply: A. None because this bilirubin level is normal B. Yellowing of the skin and sclera C. Clay-colored stools D. Bluish discoloration on the flanks of the abdomen E. Dark urine F. Mental status changes

The answers are B, C, and E. This is associated with a high bilirubin level. A normal bilirubin level is 1 or less.

Which of the following is NOT a common source of transmission for Hepatitis A? Select all that apply: A. Water B. Food C. Semen D. Blood

The answers are C and D. The most common source for transmission of Hepatitis A is water and food.

You're providing education to a patient with an active Hepatitis B infection. What will you include in their discharge instructions? Select all that apply: A. "Take acetaminophen as needed for pain." B. "Eat large meals that are spread out through the day." C. "Follow a diet low in fat and high in carbs." D. "Do not share toothbrushes, razors, utensils, drinking cups, or any other type of personal hygiene product." E. "Perform aerobic exercises daily to maintain strength."

The answers are C and D. The patient should NOT take acetaminophen (Tylenol) due to its effective on the liver. The patient should eat small (NOT large), but frequent meals...this may help with the nausea. The patient should rest (not perform aerobic exercises daily) because this will help with liver regeneration.

What two types of cancer can atrophic gastritis lead to?

gastric cancer and gastric mucosa-associated lymphoid tissue lymphoma (MALT)

B, C, E (The patient at risk is monitored for signs and symptoms indicative of addisonian crisis, which can include shock; hypotension; rapid, weak pulse; rapid respiratory rate; pallor; and extreme weakness. Epistaxis and a bounding pulse are not symptoms or signs of an addisonian crisis.)

The nurse is caring for a patient at risk for an addisonian crisis. For what associated signs and symptoms should the nurse monitor the patient? Select all that apply. A) Epistaxis B) Pallor C) Rapid respiratory rate D) Bounding pulse E) Hypotension

Hematemesis is more common than melena with what type of ulcer?

gastric ulcer

Hemorrhage tends to occur more in older adults and with what type of ulcer?

gastric ulcer

What type of ulcer pain is located to the *left* of the midline and is *aggravated by food*?

gastric ulcer

D (Oral thyroid hormones interact with many other medications.Even in small IV doses, hypnotic and sedative agents may induce profound somnolence, lasting far longer than anticipated and leading to narcosis (stupor like condition). Furthermore, they are likely to cause respiratory depression, which can easily be fatal because of decreased respiratory reserve and alveolar hypoventilation. Antibiotics, PPIs and diuretics do not cause the same risk.)

The nurse is providing care for an older adult patient whose current medication regimen includes levothyroxine (Synthroid). As a result, the nurse should be aware of the heightened risk of adverse effects when administering an IV dose of what medication? A) A fluoroquinalone antibiotic B) A loop diuretic C) A proton pump inhibitor (PPI) D) A benzodiazepine

What type of ulcer pain typically occurs 30-60 min after a meal?

gastric ulcer

D (When over secretion of GH occurs before puberty, gigantism results.)

The nurse is reviewing a client's history which reveals that the client has had an over secretion of growth hormone (GH) that occurred before puberty. The nurse interprets this as which of the following? a) Acromegaly b) Simmonds' disease c) Dwarfism d) Gigantism

A (Some endocrine disorders are inherited or have a tendency to run in families. Therefore, it is essential to take a complete family history.)

The nurse obtains a complete family history of a client with a suspected endocrine disorder based on which rationale? a) Endocrine disorders can be inherited. b) An allergy to iodine is inherited. c) It helps determine the client's general status. d) Diet and drug histories are related to the family history.

A (The nurse should take action to prevent the patients risk for falls. Bed rest carries too many harmful effects, however, and assistive devices may or may not be necessary. Constant supervision is not normally required or practicable.)

The nurse providing care for a patient with Cushing syndrome has identified the nursing diagnosis of risk for injury related to weakness. How should the nurse best reduce this risk? A) Establish falls prevention measures. B) Encourage bed rest whenever possible. C) Encourage the use of assistive devices. D) Provide constant supervision.

B (When hypocalcemia and tetany occur after a thyroidectomy, the immediate treatment is administration of IV calcium gluconate. This has a much faster therapeutic effect than PO calcium or vitamin D supplements. PTH and levothyroxine are not used to treat this complication.)

The nurses assessment of a patient with thyroidectomy suggests tetany and a review of the most recent blood work corroborate this finding. The nurse should prepare to administer what intervention? A) Oral calcium chloride and vitamin D B) IV calcium gluconate C) STAT levothyroxine D) Administration of parathyroid hormone (PTH)

A patient with acute pancreatitis puts the call bell on to tell the nurse about an increase in pain. The nurse observes the patient guarding; the abdomen is boardlike and no bowel sounds are detected. What is the major concern for this patient? a) The patient has developed renal failure. b) The patient is developing a paralytic ileus. c) The patient requires more pain medication. d) The patient has developed peritonitis.

The patient has developed peritonitis. Abdominal guarding is present. A rigid or boardlike abdomen may develop and is generally an ominous sign, usually indicating peritonitis

Total parental nutrition (TPN) should be used cautiously in patients with pancreatitis due to which of the following? a) They are at risk for gallbladder contraction. b) They can digest high-fat foods. c) They cannot tolerate high-glucose concentration. d) They are at risk for hepatic encephalopathy.

They cannot tolerate high-glucose concentration. Total parental nutrition (TPN) is used carefully in patients with pancreatitis because some patients cannot tolerate a high-glucose concentration, even with insulin coverage. Intake of coffee increases the risk for gallbladder contraction, while intake of high protein increases risk for hepatic encephalopathy in patients with cirrhosis. Patients with pancreatitis should not be given high-fat foods because they are difficult to digest.

The physician has written the following orders: bed rest, nothing by mouth (NPO), and administration of total parenteral nutrition (TPN) for a new patient admitted with pancreatitis. The nurse attributes which of the following as the cause for NPO status? a) To aid opening up of pancreatic duct b) To prevent the occurrence of fibrosis c) To drain the pancreatic bed d) To avoid inflammation of the pancreas

To avoid inflammation of the pancreas Pancreatic secretion is increased by food and fluid intake and may cause inflammation of the pancreas.

The nurse is caring for a patient with acute pancreatitis. The patient has an order for an anticholinergic medication. The nurse explains that the patient will be receiving that medication for what reason? a) To depress the central nervous system and increase the pain threshold b) To relieve nausea and vomiting c) To decrease metabolism d) To reduce gastric and pancreatic secretions

To reduce gastric and pancreatic secretions Anticholinergic medications reduce gastric and pancreatic secretion.

A patient is admitted to the hospital with possible cholelithiasis. What diagnostic test of choice will the nurse prepare the patient for? a) Cholecystography b) Ultrasonography c) Oral cholecystography d) X-ray

Ultrasonography Ultrasonography has replaced cholecystography (discussed later) as the diagnostic procedure of choice because it is rapid and accurate and can be used in patients with liver dysfunction and jaundice. It does not expose patients to ionizing radiation.

C (The thyroid concentrates iodine from food and uses it to synthesize thyroxine (T4) and triiodothyronine (T3). These two hormones regulate the body's metabolic rate.)

Wendy Corcoran, a 34-year-old teacher, is being seen at the primary care group where you practice nursing. She is undergoing diagnostics for an alteration in thyroid function. What physiologic function is affected by her altered thyroid function? a) Growth b) Sleeping, wake cycles c) Metabolic rate d) Fluid, electrolyte balance

A (An autonomous aldosterone-producing adenoma is the most common cause of hyperaldosteronism. Hyperplasia is the second most frequent cause.)

What is the most common cause of hyperaldosteronism? a) An adrenal adenoma b) Excessive sodium intake c) A pituitary adenoma d) Deficient potassium intake

B (The patient and family should be informed that acute adrenal insufficiency and underlying symptoms will recur if corticosteroid therapy is stopped abruptly without medical supervision. The patient should be instructed to have an adequate supply of the corticosteroid medication always available to avoid running out. Doses should not be skipped or added without explicit instructions to do so. Corticosteroids should normally be taken in the morning to mimic natural rhythms.)

What should the nurse teach a patient on corticosteroid therapy in order to reduce the patients risk of adrenal insufficiency? A) Take the medication late in the day to mimic the bodys natural rhythms. B) Always have enough medication on hand to avoid running out. C) Skip up to 2 doses in cases of illness involving nausea. D) Take up to 1 extra dose per day during times of stress.

When is aldosterone secreted by the adrenal cortex?

When sodium levels in the extracellular fluid are low. (Aldosterone also promotes kidney potassium excretion)

B (Hashimoto's thyroiditis, an autoimmune disorder, is the most common cause of hypothyroidism. It's seen most frequently in women older than age 40. Signs and symptoms include weight gain, decreased appetite; constipation; lethargy; dry cool skin; brittle nails; coarse hair; muscle cramps; weakness; and sleep apnea.)

Which findings should a nurse expect to assess in client with Hashimoto's thyroiditis? a) Weight loss, increased appetite, and hyperdefecation b) Weight gain, decreased appetite, and constipation c) Weight loss, increased urination, and increased thirst d) Weight gain, increased urination, and purplish-red striae

C (Parathormone (parathyroid hormone), the protein hormone produced by the parathyroid glands, regulates calcium and phosphorous metabolism.)

Which glands regulate calcium and phosphorous metabolism? a) Thyroid b) Adrenal c) Parathyroid d) Pituitary

C (During physical examination, the nurse examines the shape and color of the nails and determines whether they are thin, thick, or brittle.)

Which of the following assessments are done by the nurse when conducting a physical examination? a) Palpate the thyroid gland repeatedly b) Examine outstretched hands for skin breaks c) Examine the shape and color of the nails d) Determine the patient's ability to participate in the test

A (A computed tomography or magnetic resonance imaging scan is done to detect a suspected pituitary tumor.)

Which of the following diagnostic tests are done to determine suspected pituitary tumor? a) A computed tomography scan b) Measuring blood hormone levels c) Radiographs of the abdomen d) A radioimmunoassay

D (The patient with Cushing syndrome demonstrates truncal obesity, moon face, acne, abdominal striae, and hypertension. Regardless of the cause, the normal feedback mechanisms that control the function of the adrenal cortex become ineffective, and the usual diurnal pattern of cortisol is lost. The signs and symptoms of Cushing syndrome are primarily a result of the oversecretion of glucocorticoids and androgens, although mineralocorticoid secretion also may be affected.)

Which of the following disorders is characterized by a group of symptoms produced by an excess of free circulating cortisol from the adrenal cortex? a) Graves' disease b) Addison's disease c) Hashimoto's disease d) Cushing syndrome

D (Commonly referred to as the master gland, the pituitary gland secretes hormones that control the secretion of additional hormones by other endocrine glands.)

Which of the following glands is considered the master gland? a) Parathyroid b) Adrenal c) Thyroid d) Pituitary

Which nursing action is most appropriate for a client hospitalized with acute pancreatitis? a) Administering meperedine, as ordered, to relieve severe pain b) Withholding all oral intake, as ordered, to decrease pancreatic secretions c) Limiting I.V. fluids, as ordered, to decrease cardiac workload d) Keeping the client supine to increase comfort

Withholding all oral intake, as ordered, to decrease pancreatic secretions The nurse should withhold all oral intake to suppress pancreatic secretions, which may worsen pancreatitis.

adiponectin

a peptide that increases insulin sensitivity, decreased in obese people

What is the *early* pathologic manifestation of gastritis?

a thickened, reddened mucous membrane with prominent *rugae*

Patients diagnosed with esophageal varices are at risk for hemorrhagic shock. Which of the following is a sign of potential hypovolemia? a) Hypotension b) Bradycardia c) Polyuria d) Warm moist skin

a) Hypotension

When taking history of a client with a gastric ulcer, which assessment findings does the nurse expect? (SATA) a) vomiting b) weight loss c) epigastric pain at night d) relief of epigastric pain after eating e) melena

a, c, e

The patient with cirrhosis is being taught self-care. Which statement indicates the patient needs more teaching? a. "If I notice a fast heart rate or irregular beats, this is normal for cirrhosis." b. "I need to take good care of my belly and ankle skin where it is swollen." c. "A scrotal support may be more comfortable when I have scrotal edema." d. "I can use pillows to support my head to help me breathe when I am in bed."

a. "If I notice a fast heart rate or irregular beats, this is normal for cirrhosis."

The nurse is caring for a woman recently diagnosed with viral hepatitis A. Which individual should the nurse refer for an immunoglobulin (IG) injection? a. A caregiver who lives in the same household with the patient b. A friend who delivers meals to the patient and family each week c. A relative with a history of hepatitis A who visits the patient daily d. A child living in the home who received the hepatitis A vaccine 3 months ago

a. A caregiver who lives in the same household with the patient

a patient with hep. A is in the acute phase. the nurse plans care for the patient based on the knowledge that a. pruritus is a common problem with jaundice in this phase. b. the patient is most likely to transmit the disease during this phase. c. GI symptoms are not as severe in hep A as they are in hep B d. extrahepatic manifestations of glomerulonephritis and plyarteritis are common in this phase

a. pruritus is a common problem with jaundice in this phase.

Describe the pain/signs associated with perforation.

abdomen is *tender*, *rigid*, and boardlike (peritonitis) -patient assumes fetal position to decrease tension -severely ill within hours -bacterial septicemia and hypovolemic shock follow -peristalsis diminishes -paralytic ileus develops

Inflammation in the gastric mucosa after *exposure to local irritants* can result in what type of gastritis?

acute gastritis

Long-term NSAID use creates a high risk for what type of gastritis?

acute gastritis (*NSAIDs ihibit prostaglandin production*)

The digestion of carbohydrates is aided by a) trypsin. b) secretin. c) amylase. d) lipase.

amylase. Amylase is secreted by the exocrine pancreas. Lipase aids in the digestion of fats. Trypsin aids in the digestion of proteins. Secretin is the major stimulus for increased bicarbonate secretion from the pancreas.

fat in abdominal area

android obesity

What type of drugs are used as *buffering agents* for patients with gastritis?

antacids

Which drug class *increases pH of gastric contents by deactivating pepsin*?

antacids (*give 2 hr after meal and at bedtime*) (*do not give other drugs within 1-2 hr of antacids*) *assess pt. history of RENAL disease and HEART FAILURE*

Where do gastric ulcers typically develop?

antrum of the stomach (*lesser curvature of the stomach near the pylorus*) near acid-secreting mucosa

■The nurse has identified the nursing diagnosis of fatigue for a client who is hypothyroid. What should the nurse do well caring for this client? -A. Monitor for changes in orientation, cognition, and behavior. -B. Monitor for vital signs in cardiac rhythm response to activity. -C. Monitor bowel movement frequency, consistency, shaped, volume, and color. -D. Assist in developing well-balanced meal plans consistent with energy expenditure level.

b

■A client with hypothyroidism is treated with levothyroxine. What should the nurse include when teaching the client about this therapy? -A. Explain the alternate day dosage may be used if side effects occur. -B. Provide written instructions for all information related to the drug therapy. -C. Assure the client that a return to normal function will occur with replacement therapy. -D. Inform the client that the drug must be taken until the hormone balance is reestablished.

b because of mental sluggishness due to hypothyroidism

What type of ulcers are associated with *normal gastric acid secretion*, *delayed gastric emptying*, and *increased diffusion of gastric acid back into the stomach tissues*?

gastric ulcers

The nurse is aware of potential complications related to cirrhosis. Which interventions would be included in a safe plan of care (select all that apply.)? a. Provide a high-protein, low-carbohydrate diet. b. Teach the patient to use soft-bristle toothbrush and electric razor. c. Teach the patient to avoid vigorous blowing of nose and coughing. d. Apply gentle pressure for the shortest possible time after venipuncture. e. Use the smallest gauge needle possible when giving injections or drawing blood. f. Instruct the patient to avoid aspirin and nonsteroidal antiinflammatory (NSAIDs).

b. Teach the patient to use soft-bristle toothbrush and electric razor. c. Teach the patient to avoid vigorous blowing of nose and coughing. e. Use the smallest gauge needle possible when giving injections or drawing blood. f. Instruct the patient to avoid aspirin and nonsteroidal antiinflammatory (NSAIDs).

the patient with advanced cirrhosis asks why his abdomen is so swollen. the nurse's response is based on the knowledge that a. a lack of clotting factors promotes the collection of blood in the abdominal cavity. b. portal hypertension and hypoalbuminemia cause a fluid shift into the peritoneal space c. decreased peristalsis in the GI tract contributes to gas formation and distention of the bowel d. bile salts in the blood irritate the peritoneal membranes, causing edema and pocketing of fluid

b. portal hypertension and hypoalbuminemia cause a fluid shift into the peritoneal space

a patient has been told that she has elevated liver enzymes caused by NAFLD. the nursing teaching plan should include a. having genetic testing done b. recommending a heart-healthy diet c. the necessity to reduce weight rapidly d. avoiding alcohol until liver enzymes return to normalb

b. recommending a heart-healthy diet

a patient with acute hep. b is being discharged in 2 days. the discharge teaching plan should include instructions to a. avoid alcohol for first 3 weeks b. use condoms c. have family members get an injection of immunoglobulin d. follow a low-protein, moderate-carbohydrate, moderate0fat diet

b. use condoms

what is a sengstaken-blackmore tube-balloon tamponade

balloon that puts pressure on esophagus to help control bleeding the balloon inflates in the stomach and in the esophagus so it does not move and cause it to obstruct the airway deflate the esophagus balloon first then the stomach so that it does not obstruct the airway while removing

What is inserted into the patient's mouth to protect the teeth and prevent biting down on the endoscope during an EGD?

bite block

What is the main manifestation of acute stress ulcers?

bleeding caused by gastric erosion

Curling's ulcer is associated with what?

burns

What is the inflammation of the gastric mucosa that can be localized or diffused and erosive or nonerosive?

gastritis

A nurse is caring for a client with cirrhosis. The nurse assesses the client at noon and discovers that the client is difficult to arouse and has an elevated serum ammonia level. The nurse should suspect which situation? a) The client didn't take his morning dose of lactulose (Cephulac). b) The client is relaxed and not in pain. c) The client's hepatic function is decreasing. d) The client is avoiding the nurse.

c) The client's hepatic function is decreasing.

The nurse has been assigned to provide care for 4 clients at the beginning of the day shift. In what order does the nurse assess these clients? a) a client planned for EGD at 1 pm b) a client requesting pain meds 2 days after partial gastrectomy c) a client with PUD experiencing a sudden onset of acute stomach pain d) a client who is NPO for tests to rule out gastric cancer

c, b, a, d

The nurse provides discharge instructions for a 64-yr-old woman with ascites and peripheral edema related to cirrhosis. Which patient statement indicates teaching was effective? a. "Lactulose should be taken every day to prevent constipation." b. "It is safe to take acetaminophen up to four times a day for pain." c. "Herbs and other spices should be used to season my foods instead of salt." d. "I will eat foods high in potassium while taking spironolactone (Aldactone)."

c. "Herbs and other spices should be used to season my foods instead of salt."

Which antacid triggers gastrin release which causes rebound acid secretion, therefore is not recommended for acid inhibition?

calcium carbonate (TUMS)

BMI

calculated by dividing a person's weight byt the square of the height in meters

When does obesity typically become a problem?

childhood and adolescents years

Reversing ____________________ is the key to addressing the overall obesity epidemic.

childhood obesity

What type of gastritis appears as a *patchy, diffuse inflammation* of the mucosal lining of the stomach?

chronic gastritis

With what type of gastritis is the *function of the parietal cells (acid-secreting) decreased* and the *source of intrinsic factor lost* (therefor associated with the development of *pernicious anemia*)?

chronic gastritis

greater than 40 in men greater than 35 in women

considered increased for health risk

what does vasopressin do in a patient with Esophageal Varices

constricts the vessels

Gallstones

crystallized cholesterol in the gallbladder

Increased intracranial pressure is associated with what type of ulcer?

cushing's ulcers

■What is the cause of primary hypothyroidism in adults? -A. Malignant or benign thyroid nodules. -B. Surgical removal or failure of the pituitary gland. -C. Surgical removal or radiation of the thyroid gland. -D. Autoimmune-induced atrophy of the thyroid gland

d

■When replacement therapy is started for a client with long-standing hypothyroidism, what is the most important for the nurse to monitor for in the client? -A. Insomnia -B. Weight loss -C. Nervousness -D. Dysrhythmias

d

how do we give blood products with Esophageal Varices

given very quickly and warmed so we do not cause hypothermia in our patients

The family of a patient newly diagnosed with hepatitis A asks the nurse what they can do to prevent becoming ill. Which response by the nurse is most appropriate? a. "You will need to be tested first; then treatment can be determined." b. "The hepatitis vaccine will provide immunity from this and future exposures." c. "There is nothing you can do since the patient was infectious before admission." d. "An immunoglobulin injection will be given to prevent infection or limit symptoms."

d. "An immunoglobulin injection will be given to prevent infection or limit symptoms."

When teaching the patient with acute hepatitis C (HCV), which statement demonstrates understanding of the disease process? a. "I will use care when kissing my wife to prevent giving it to her." b. "I will need to take adefovir (Hepsera) to prevent chronic HCV." c. "Now that I have had HCV, I will have immunity and not get it again." d. "I will need to be monitored for chronic HCV and other liver problems."

d. "I will need to be monitored for chronic HCV and other liver problems."

A patient with hepatitis B surface antigen (HBsAg) present in the serum is being discharged with pain medication after knee surgery. Which medication order should the nurse question? a. Tramadol b. Hydromorphone (Dilaudid) c. Oxycodone with aspirin (Percodan) d. Hydrocodone with acetaminophen

d. Hydrocodone with acetaminophen

When planning care for a patient with cirrhosis, the nurse will give highest priority to which nursing diagnosis? a. Impaired skin integrity related to edema, ascites, and pruritus b. Imbalanced nutrition: less than body requirements related to anorexia c. Excess fluid volume related to portal hypertension and hyperaldosteronism d. Ineffective breathing pattern related to pressure on diaphragm and reduced lung volume

d. Ineffective breathing pattern related to pressure on diaphragm and reduced lung volume

A client with acute hepatitis is prescribed lactulose. The nurse knows this medication will: a. Mobilize iron stores from the liver. b. Remove bilirubin from the blood. c. Prevent hypoglycemia. d. Prevent the absorption of ammonia from the bowel.

d. Prevent the absorption of ammonia from the bowel.

The health care provider orders lactulose for a patient with hepatic encephalopathy. Which finding indicates the medication has been effective? a. Relief of constipation b. Relief of abdominal pain c. Decreased liver enzymes d. decreased ammonia levels

d. decreased ammonia levels

What happens to gastric emptying in patients with gastric ulcers?

delayed

Obesity is considered a ____________ and _____________.

disease and public health crisis

Interventions to manage the pain involved with PUD focus on what?

drug therapy and diet changes

Melena is more common in patients with what type of ulcer?

duodenal ulcer

Melena is more common than hematemesis with what type of ulcer?

duodenal ulcer

What type of ulcer pain is *relieved by the ingestion of food*?

duodenal ulcer

What type of ulcer pain is located to the *right* of or below epigastrium and occurs *90 min to 3 hours after eating* and often awakens the person at night?

duodenal ulcer

Which type of peptic ulcer usually occurs in patients with *type O blood*?

duodenal ulcer

What type of ulcers are associated with *increased secretion of gastric acid*, *increased gastric emptying*, and *normal diffusion of acid back into stomach tissues*?

duodenal ulcers

A client with cholelithiasis has a gallstone lodged in the common bile duct. When assessing this client, the nurse expects to note: a) circumoral pallor. b) yellow sclerae. c) black, tarry stools. d) light amber urine.

yellow sclerae. Yellow sclerae are an early sign of jaundice, which occurs when the common bile duct is obstructed. Urine normally is light amber. Circumoral pallor and black, tarry stools don't occur in common bile duct obstruction; they are signs of hypoxia and GI bleeding, respectively.

For an EGD is the head of the bed elevated or no?

yes

The nurse admits a woman who is 8 weeks postpartum. The client presents with reports of severe right upper quadrant pain that radiates to her back. She states she just finished eating Thanksgiving dinner earlier that evening. The nurse suspects gallbladder disease. Statistics show that incidence of gallbladder disease is greater for women who are which of the following? Select all that apply. a) Multiparous b) Obese c) Thin d) Wealthy e) Older than 60 years

• Multiparous • Obese Two to three times more women than men develop cholesterol stones and gallbladder disease; affected women are usually older than 40 years, multiparous, and obese.

When caring for the patient with acute pancreatitis, the nurse must consider pain relief measures. What nursing interventions could the nurse provide? (Select all that apply.) a) Withholding oral feedings to limit the release of secretin b) Administering parenteral opioid analgesics as ordered c) Administering prophylactic antibiotics d) Encouraging bed rest to decrease the metabolic rate e) Assisting the patient into the prone position

• Withholding oral feedings to limit the release of secretin • Administering parenteral opioid analgesics as ordered • Encouraging bed rest to decrease the metabolic rate The current recommendation for pain management is the use of opioids, with assessment for their effectiveness and altering therapy if pain is not controlled or increased (Marx, 2009). Nonpharmacologic interventions such as proper positioning (not prone), music, distraction, and imagery may be effective in reducing pain when used along with medications. In addition, oral feedings are withheld to decrease the secretion of secretin.

What lab values are seen in hypothyroidism?

↑ Cholesterol ↑ Triglycerides ↑ Creatine kinase ↓ RBCs (anemia) decreased T4


Related study sets

Care Coordination and other EAQ Questions

View Set

Social changes: positive contributions

View Set

First 30 Elements & their Symbols

View Set

Chemistry Test 4- Evidence of Chemical Reactions 1

View Set